Gen Emergency Surgery Flashcards

1
Q
1. An 85-year-old woman presents with a 6-hour history of severe sudden onset central abdominal pain. Her heart rate is 110 beats per minute and irregularly irregular. Her blood pressure is 132/94 mmHg and temperature is 38.1°C. Abdominal examination reveals a generally soft but distended and tender abdomen with no audible bowel sounds. Per rectal examination reveals guaiac-positive stool.
What is the most likely diagnosis?
A Abdominal aortic aneurysm
B Acute pancreatitis
C Mesenteric ischaemia
D Myocardial infarction
E Peptic ulcer disease
A
  1. C Mesenteric ischaemia
    Acute mesenteric ischaemia is a serious condition with a high morbidity and mortality. It can also be notoriously difficult to diagnose. CT of the abdomen with arterial contrast may show thickened loops of small bowel or thrombus in the involved arteries (coeliac axis, superior mesenteric artery or inferior mesenteric artery) (Figure 1.1).
    Figure 1.1 CT of small bowel ischaemia.
    A 10-year retrospective review of cases in the USA revealed 95% of cases presented with abdominal pain, and 44% had nausea. Vomiting and diarrhoea occurred in 35% of cases, tachycardia in 33%, and 16% had per rectal bleeding. Atrial fibrillation is strongly associated with mesenteric ischaemia as embolic cause accounts for 28% of cases and half of these patients are fibrillating.
    Park WM, Gloviczki P, Cherry KJ Jr, et al. Contemporary management of acute mesenteric ischemia: Factors associated with survival. J Vasc Surg 2002; 35:445–452.
    Answers 31
How well did you know this?
1
Not at all
2
3
4
5
Perfectly
2
Q
  1. In patients who present with acute pancreatitis, which factor is least likely to independently predict the development of complications?

A Age > 55 years
B APACHE II score > 8 points in the first 24 hours of admission
C C-reactive protein > 150 mg/L after 48 hours in hospital
D Glasgow score > 3
E Obesity

A
  1. A Age > 55 years
    Age above 55 years alone is not predictive of the development of complications. Complications in acute pancreatitis occur more commonly in patients with clinical obesity. Complications are also more common in those patients with severe pancreatitis. This can be based on scoring systems (an APACHE II score > 8 in the first 24 hours after admission or a Glasgow coma score > 3). Severe pancreatitis also includes any organ failure which continues beyond 48 hours and a C-reactive protein > 150 mg/L. In both cases complications are more likely to occur.
    UK Working Party on Acute Pancreatitis. UK guidelines for the management of acute pancreatitis. Gut 2005; 54:1–9.
How well did you know this?
1
Not at all
2
3
4
5
Perfectly
3
Q
  1. An 8-week-old infant is diagnosed with a congenital diaphragmatic hernia (CDH). What is the most common cause of a CDH?
    A Anterior defect (Morgagni hernia)
    B Central tendon defect
    C Defect in the pleuroperitoneal fold
    D Enlarged oesophageal hiatus
    E Posterolateral defect (Morgagni hernia)
A
  1. C A defect in the pleuroperitoneal fold
    A defect in the pleuroperitoneal fold may result in a posterolateral Bochdalek hernia which is the most common congenital diaphragmatic hernia, accounting for more than 95% of cases. The majority of Bochdalek hernias (78%) occur on the left side of the diaphragm, 20% on the right, and they are rarely bilateral. Unlike Morgagni herniae, Bochdalek herniae tend not to have a hernia sac. Morgagni herniae occur through the anterior space of Larrey.
    Brown SR, Horton JD, Trivette E, et al. Bochdalek hernia in the adult: demographics, presentation, and surgical management. Hernia 2011; 15:23–30.
How well did you know this?
1
Not at all
2
3
4
5
Perfectly
4
Q
  1. A 27-year-old man presents with colicky pain in his left flank and urine dipstick reveals the presence of haematuria. A plain abdominal radiograph reveals a
  2. 5 × 0.5 cm opacification in his bladder which is thought to represent a passed calculus.

What is this calculus most likely to be composed of?
A Calcium oxalate
B Calcium phosphate
C Mixed oxalate/phosphate
D Ammonium magnesium phosphate (struvite)
E Uric acid

A
  1. A Calcium oxalate
    Statistically, this stone is most likely to be composed of calcium oxalate as this comprises 40% of ureteric calculi. Mixed oxalate/phosphate occurs in 20%, calcium phosphate and ammonium magnesium phosphate (struvite) stones both occur in 15% of patients and uric acid in 10%.
    Stones containing calcium, such as calcium oxalate and calcium phosphate, are easiest to detect by radiography. Although 90% of urinary calculi have been considered to be radiopaque, the sensitivity and specificity of kidney, ureters, and bladder radiography alone remains poor (with sensitivity of 45–59% and specificity of 71–77%).
    Levine JA, Neitlich J, Verga M, et al. Ureteral calculi in patients with flank pain: correlation of plain radiography with unenhanced helical CT. Radiology 1997; 204:27–31.
How well did you know this?
1
Not at all
2
3
4
5
Perfectly
5
Q
  1. A 6-year-old boy with abdominal pain is admitted to the paediatric surgical ward for investigation and treatment. He is known from a previous admission to have a Meckel’s diverticulum and his attending doctors wonder if it is the cause of his current symptoms.
    Which of the following is not a recognised presentation of a Meckel’s diverticulum?
    A Gastrointestinal haemorrhage
    B Incidental mesenteric lesion found at laparoscopy/laparotomy
    C Intussusception
    D Perforation
    E Symptomatic inguinal hernia
A
  1. B Incidental mesenteric lesion found at laparoscopy/laparotomy
    Meckel’s diverticulae are classically 5 cm (2 inch) long pulsion diverticulae which are normally found approximately 60 cm (2 feet) from the ileocaecal valve on the antimesenteric border of the ileum. They occur in approximately 2% of the population and hence the ‘Rule of 2s applies’. They represent an embryological remnant of the vitello-intestinal duct. Most patients are asymptomatic although the Meckel’s divertican cause symptoms due to inflammation, bleeding or perforation. It can also present as the lead point for an intussusception or as the contents of an indirect inguinal hernia sac, in the eponymous Littre’s hernia.
    Thurley PD, Halliday KE, Somers JM, et al. Radiological features of Meckel’s diverticulum and its complications. Clin Radiol 2009; 64:109–118.
How well did you know this?
1
Not at all
2
3
4
5
Perfectly
6
Q
  1. A 3-week-old neonate presents with dehydration and bilious vomiting. A contrast study reveals a duodenojejunal junction lying on the right side of the abdomen.
    What is the most appropriate definitive management?
A Hydrostatic reduction
B Insertion of nasogastric tube
C Intravenous fluids and antiemetics
D Laparotomy
E Review at 6 months of age to determine the need for intervention
A
  1. D Laparotomy
    This patient has intestinal malrotation. This is a congenital condition occurring 1 in 500 live births, which can have serious complications. Malrotation occurs when there is a failure of embryological gut development and rotation, whereby the root of the mesentery remains narrow and the caecum occupies a non-rotated position in the right upper quadrant

In the normal physiological state, starting from a straight tube, the embryological gut undergoes a complex series of rotations between 4–12 weeks culminating in the duodenojejunal flexure lying on the left side of the abdomen and the caecum positioned in the right iliac fossa.
There is a subsequent tendency for the bowel to twist around the mesentery and this combined with the development of adhesive Ladd’s bands can lead to ischaemic bowel and perforation. As such early diagnosis is important to facilitate early laparotomy and avoid dangerous sequelae.

Operative repair involves division of the Ladd’s band, appendicectomy, widening of the mesentery and relocation of the colon. A laparoscopic form of this procedure has evolved with the contrived acronym ‘LADD’ (Laparoscopic Appendicectomy and Duodenocolic Dissociation).
Millar AJW, Rode H, Brown RA, Cywes S. The deadly vomit: malrotation and midgut volvulus. Pediatr Surg Int 1987; 2:172–176.
Lessin MS, Luks FI. Laparoscopic appendectomy and duodenocolonic dissociation (LADD) procedure for malrotation. Pediatr Surg Int 1998; 13:184–185.
Williams H. Green for danger! Intestinal malrotation and volvulus. Arch Dis Child Educ Pract Ed 2007; 92:87–91.

How well did you know this?
1
Not at all
2
3
4
5
Perfectly
7
Q
  1. A 4-year-old infant is diagnosed with a symptomatic choledochal cyst.
    What is the most appropriate treatment?
A Cyst gastrostomy
B Cyst jejunostomy
C Excision with Roux-en-Y hepaticojejunostomy
D Primary excision of the lesion
E Surveillance (endoscopic)
A
  1. C Excision with Roux-en-Y hepaticojejunostomy
    Choledochal cysts are congenital dilations of the intra- and/or extrahepatic bile duct(s). They can be asymptomatic or present with components of the classic triad of jaundice, abdominal pain and right upper quadrant mass.
    They can present in children or adults and have been classified by Todani et al (1977) into types I–V (Figure 1.3):
    • Type I: dilation of all or part of common bile duct (CBD; this is the commonest form, representing 80–90% of all choledochal cysts).
    • Type II: solitary cystic diverticulum extending from the CBD.
    • Type III: arising from the duodenal CBD at its junction with the pancreatic duct.
    • Type IV: cystic dilatations of the intra- and extrahepatic biliary tree.
    • Type V: cystic dilatations of the intrahepatic biliary tree only.
    Children with choledochal cysts fare differently from adults as long-term complications are less common. The management of choledochal cysts used to involve internal drainage with cystoduodenostomy or cystoduodenotomy, but these procedures had a tendency to fail and up to 5% developed adenocarcinoma. The surgical treatment of choice is excision of the lesion and formation of a hepaticojejunostomy.
    Todani T, Watanabe Y, Narusue M, et al. Congenital bile duct cysts: Classification, operative procedures, and review of thirty-seven cases including cancer arising from choledochal cyst. Am J Surg 1977; 134:263–269.
    Shimotakahara A, Yamataka A, Yanai T, et al. Roux-en-Y hepaticojejunostomy or hepaticoduodenostomy for biliary reconstruction during the surgical treatment of choledochal cyst: which is better? Pediatr Surg Int 2005; 21:5–7.
How well did you know this?
1
Not at all
2
3
4
5
Perfectly
8
Q
  1. A 21-year-old previously healthy male arts student presents with a 2-day history of lower abdominal pain and loss of appetite. He is pyrexial and has a tender palpable mass in his right iliac fossa. Haematology reveals a pyogenic leukocytosis and a CT demonstrates a 13 × 15 cm walled-off abscess adjacent to a large complex inflammatory mass involving an inflamed appendix.

What is the most appropriate management plan?
A Conservative management (analgesia, intravenous fluid and antibiotics)
B Conservative management, followed by interval appendicectomy at 6–8 weeks
C Laparoscopy and washout
D Laparoscopic appendicectomy
E Percutaneous drainage of the abscess and intravenous antibiotics

A
  1. E Percutaneous drainage of abscess and intravenous antibiotics
    In patients with an appendix mass there is a degree of clinical equipoise with regards to the specific management. The main treatment decisions are between:
    • Immediate appendicectomy
    • Conservative management with interval appendicectomy
    • Conservative management alone.
    The presence of an appendix abscess mandates either surgical or radiological drainage. In this patient’s case there is a complex inflammatory mass and, as such, a surgical procedure would not be advisable.
    Paull DL. Bloom GP. Appendiceal abscess. Arch Surg 1982; 117:1017–1019.
    Andersson RE, Petzold MG. Nonsurgical treatment of appendiceal abscess or phlegmon: a systematic review and meta-analysis. Ann Surg 2007; 246:741–748.
    Brown CV, Abrishami M, Muller M, Velmahos GC. Appendiceal abscess: immediate operation or percutaneous drainage? Am Surg 2003; 69:829–832.
    Lasson A, Lundagårds J, Lorén I, Nilsson PE. Appendiceal abscesses: primary percutaneous drainage and selective interval appendicectomy. Eur J Surg 2002;168:264–269.
How well did you know this?
1
Not at all
2
3
4
5
Perfectly
9
Q
  1. A 70-year-old man with arteriopathy undergoes a barium enema and feels light-headed afterwards. Routine biochemistry reveals a serum potassium of 2.8 mmol/L.
    Which medication is least likely to be causative?
A Amphotericin B
B Barium
C Furosemide
D Salbutamol
E Trimethoprim
A
  1. E Trimethoprim
    Trimethoprim is associated with hyperkalaemia. It inhibits the activity of the epithelial sodium channel resulting in elevated serum potassium. The four other drugs listed here as answer options are all associated with hypokalaemia rather than hyperkalaemia. Exposure to barium in the enema, antifungal treatment with amphotericin, diuretic therapy such as furosemide, and inhalers such as salbutamol, may all cause low potassium.
    Perazella MA. Trimethoprim Is a potassium-sparing diuretic like amiloride and causes hyperkalemia in high-risk patients. Am J Therapeutics 1997; 4:343–8.
    Nyirenda MJ, Tang JI, Padfield PL, Seckl JR. Hyperkalaemia BMJ 2009; 339:1019–1024.
    Answers 35
How well did you know this?
1
Not at all
2
3
4
5
Perfectly
10
Q
  1. A 45-year-old woman with haematemesis is admitted to accident and emergency in hypovolaemic shock. She undergoes resuscitation including administration of packed red cells. The blood transfusion centre will not release certain blood products unless a ‘massive bleeding’ protocol is initiated.

Which of the following is not a definition of massive bleeding?

A Blood loss of half the patient’s circulating volume in a 3-hour period
B Blood loss of the patient’s circulating volume in a 24-hour period
C Ongoing blood loss of 100 mL/min
D Transfusion of 4 units of red cells in 4 hours with continued bleeding
E Transfusion of 10 units of packed red cells in a 24-hour period

A
  1. C Ongoing blood loss of 100 mL/min
    Massive bleeding is defined as:
    • Blood loss of half the patient’s circulating volume in a 3-hour period
    • Blood loss of the patient’s circulating volume in a 24-hour period
    • Ongoing blood loss of > 150 mL/min
    • Transfusion of four units of packed red cells in a 4-hour period with continued
    bleeding
    • Transfusion of 10 units of packed red cells in a 24-hour period.
    A patient meeting such criteria should stimulate the activation of the local ‘massive transfusion protocol’. This should alert the transfusion service to provide prompt delivery of packed red cells and suitable clotting products.
    British Committee for Standards in Haematology, Stainsby D, MacLennan S, et al. Guidelines on the management of massive blood loss. Br J Haematol 2006; 135:634–41.
    Riskin DJ, Tsai TC, Riskin L, et al. Massive transfusion protocols: the role of aggressive resuscitation versus product ratio in mortality reduction. J Am Coll Surg 2009; 209:198–205.
How well did you know this?
1
Not at all
2
3
4
5
Perfectly
11
Q
  1. A 43-year-old previously well woman presents with pain, swelling and erythema in the anorectal area.
    What is the most appropriate management?
A CT of abdomen and pelvis
B Examination under anaesthesia and drainage of pus
C Flexible sigmoidoscopy
D Intravenous antibiotics
E MRI of perineum
A
  1. B Examination under anaesthesia (EUA) and drainage of pus
    Anorectal sepsis is a common presenting symptom which can present acutely as an abscess with erythema, pain and swelling or chronically as a fistula-in-ano. The peak incidence of perianal abscesses are in the third decade. Management of a perianal abscess is of simple incision and drainage of underlying pus, whereas fistulotomy of a low fistula or insertion of a draining seton can be used if a fistula-in-ano is evident.
    MacKay GJ, Dorrance HR, Molloy RG (eds). Colorectal Surgery. Oxford: Oxford University Press, 2010: 274. Philips RKS (ed). Colorectal Surgery: A Companion to Specialist Surgical Practice, 4th edn. Philadelphia: Saunders, Elsevier, 2009: 227.
How well did you know this?
1
Not at all
2
3
4
5
Perfectly
12
Q
  1. A 65-year-old man with severe pancreatitis is intubated and ventilated in the intensive care unit. His intra-abdominal pressure is measured using a catheter in his bladder connected to manometry.
    Which one of the following describes the pressure effect seen in abdominal compartment syndrome?

A Bladder pressure of 16–25 mmHg does not present with oliguria
B Bladder pressure of 16–25 mmHg does not require decompression
C Bladder pressure of 26–35 mmHg results in increased cardiac output
D Bladder pressure of > 35 mmHg does not cause anuria
E Bladder pressure of < 15 mmHg normally has clinical signs

A
  1. B A bladder pressure of 16–25 mmHg does not require decompression

As intra-abdominal pressure rises, so too does bladder pressure. Between 16 and 25 mmHg is grade 2 intra-abdominal hypertension, which normally manifests as oliguria. Recommended management at this stage includes fluid resuscitation and if the pressure rises beyond the critical threshold of 25 mmHg, abdominal decompression is required (Table 1.1).

Papavramidis TS, Marinis AD, Pliakos, et al. Abdominal compartment syndrome – Intra-abdominal hypertension: Defining, diagnosing, and managing. J Emerg Trauma Shock 2011; 4:279–291

How well did you know this?
1
Not at all
2
3
4
5
Perfectly
13
Q
  1. An 84-year-old man presents repeatedly with abdominal distension due to a sigmoid volvulus.
    Which one of the following statements does not apply to the management of sigmoid volvulus?

A Following detorsion, volvulus recurs in 50–90% of cases
B Following sigmoid resection, volvulus recurs in 0% of cases
C Sigmoidoscopic decompression successfully reduces volvulus in 70–80% of
cases
D Spontaneous detorsion of volvulus is common
E Therapeutic barium enema can successfully reduce a volvulus

A
  1. D Spontaneous detorsion of volvulus is common
    Sigmoid volvulus has a characteristic inverted U-shape on plain abdominal X-ray (Figure 1.4). The management of sigmoid volvulus is relief of the obstruction, normally by sigmoidoscopic decompression. This technique is successful in 70–80% of cases. A rectal tube can be left in situ to reduce the chance of recurrence. Despite a successful result, volvulus recurs in 50–90% of patients, and as such this technique should be considered as a temporising measure only.
    Following further medical assessment and resuscitation, a decision can be made to proceed to definitive surgery (removal of the sigmoid colon). This may also be indicated if the volvulus recurs, or cannot be decompressed.
    Other therapeutic options include barium or water-soluble contrast enemas.
    This technique results in detorsion of the volvulus in approximately 5% of adults, although better results have been achieved in children. Spontaneous reductions in all patients are rare and occur in only two per cent of patients.
    Ballantyne GH. Review of sigmoid volvulus: history and results of treatment. Dis Colon Rectum 1982; 25:494–501.
    Reilly PMJ, Jones B, Bulkley GB. Volvulus of the colon. In: Cameron JL (ed). Current Surgical Therapy. St Louis: Decker Inc 1992: 170–174.
How well did you know this?
1
Not at all
2
3
4
5
Perfectly
14
Q
  1. Which of the following is most likely to shift the oxygen haemoglobin dissociation curve to the left?
A Decreased pH
B Extreme altitude
C Increased 2,3-diphosphoglycerate concentration
D Increased PaCO2
E Increased temperature
A
  1. B Extreme altitude
    The oxygen haemoglobin dissociation curve represents the relationship
    between the partial pressure of oxygen and the oxygen saturation. The affinity of haemoglobin for oxygen increases as further molecules of oxygen are bound. This results in a sigmoid-shaped curve until no further oxygen can be bound.
    When this curve is shifted to the left, this represents a higher affinity of haemoglobin for oxygen at that given pressure. Conversely, when this curve is shifted to the right this represents lower affinity and consequently oxygen is released to the tissues more readily.
    The curve is shifted to the right by acidosis (decreased pH), increased temperature, increased 2,3-diphosphoglycerate and raised CO2. This is analogous to exercising muscles and means that more oxygen is made available to the tissues. At extreme altitude the oxygen haemoglobin dissocation curve shifts to the left because there is much less CO2 in the blood (Figure 1.5).
How well did you know this?
1
Not at all
2
3
4
5
Perfectly
15
Q
  1. Prothrombin complex concentrate is a haematological product, which is used for patients with ongoing bleeding.
    Which of the following statements does not describe prothrombin complex concentrate?

A Contains clotting factors II, VII, VIII and IX
B Contains protein C and protein S
C Has a faster mode of action than fresh frozen plasma
D It is used for perioperative prophylaxis of bleeding in acquired deficiency of
the prothrombin complex coagulation factors
E Is used for reversal of warfarin when rapid correction is required

A
  1. A Contains clotting factors II, VII, VIII and IX
    Prothrombin complex concentrate contains clotting factors II, VII, IX and X (the vitamin K-dependent factors). It also contains protein C and protein S. It is indicated for treatment, and perioperative prevention, of bleeding in patients with (acquired or congenital) deficiency of factors II, VII, IX or X, or if purified specific coagulation factors are not available.
    Lubetsky A. Efficacy and safety of a prothrombin complex concentrate (Octaplex) for rapid reversal of oral
    anticoagulation. Thrombosis Research 2004; 113:371–378
How well did you know this?
1
Not at all
2
3
4
5
Perfectly
16
Q
  1. A 27-year-old woman sustains multiple rib fractures in a road traffic accident. When is an urgent thoracotomy not indicated?

A Cardiac tamponade
B Chest drainage of 300 mL blood in the first hour and 250 mL/h in the next few
hours
C Flail chest with evidence of haemothorax
D Immediate chest drainage of >1500 mL of blood
E Widening of the mediastinum to >8 cm

A
  1. C Flail chest with evidence of haemothorax
    Indications for an urgent thoracotomy include:
    • A defect in the chest wall
    • A great vessel injury demonstrated on angiography
    • A large ongoing air leak (despite adequate drainage)
    • A large unevacuated clotted haemothorax
    • A traumatic (sepal or valvular) injury to the heart
    • Cardiac tamponade
    • Chest drainage > 1500 mL (or > 200 mL drainage per hour)
    • Diaphragm laceration
    • Oesophageal injury
    • Widening of the mediastinum > 8 cm
    Bodai BI. Emergency thoracotomy in the management of trauma. JAMA 1983; 8; 249:1891–1896.
    Feliciano DV, Rozycki GS. Advances in the diagnosis and treatment of thoracic trauma. Surg Clin North Am 1999; 79:1417–1429.
    Shields, TW, Locicero J, Reed CE (eds). General Thoracic Surgery, Volume 1, 7th edn. Philadelphia: Lippincott, Williams and Wilkins 2009: 898
How well did you know this?
1
Not at all
2
3
4
5
Perfectly
17
Q
  1. A 65-year-old man presents with left iliac fossa pain, pyrexia and raised white cell count 20 × 1000 per mm3 and C-reactive protein of 200 mg/L.
    What is the most appropriate investigation?
    A Abdominal X-ray
    B Barium enema
    C Colonoscopy
    D CT of abdomen and pelvis
    E Flexible sigmoidoscopy
A
  1. D CT of abdomen and pelvis
    Diverticular disease is its complicated form can present with a patient showing clinical signs of sepsis and will require treatment with intravenous antibiotics. In severe sepsis combination antibiotics should be prescribed. In an unwell patient a CT of abdomen and pelvis can provide information as to the cause of the sepsis, i.e. confirming or refuting the suspected diagnosis as well as providing further details, such as any underlying absesses which may be ameniable to radiological drainage. A CT may also show any evidence of fistulation, obstruction or perforation, which may require more aggressive management.
    MacKay GJ, Dorrance HR, Molloy RG and O’Dwyer PJ. Colorectal Surgery. Oxford: Oxford University Press, 2010: 298.
    Philips RK. Colorectal Surgery: A Companion to Specialist Surgical Practice. Philadelphia: Saunders, 4th edn, 2009: 112.
How well did you know this?
1
Not at all
2
3
4
5
Perfectly
18
Q
  1. A 23-year-old woman is referred by the gastroenterology team with tachycardia, hypotension and generalised colonic tenderness. She has had 5 days of intravenous (IV) steroids and her bowel movements have decreased from
    12 to 6 bloody stools per day.
    What is the most appropriate management?
A Colonoscopy and biopsies
B Continue IV steroids
C Cyclosporin
D Infliximab
E Subtotal colectomy and ileostomy
A
  1. E Subtotal colectomy and ileostomy
    This patient has severe acute colitis and is systemically unwell. The treatment
    is an emergency subtotal colectomy and end ileostomy. She has failed medical management. Although there are a number of classifications for acute colitis, Truelove and Witts’ classification (Table 1.2) provides a straightforward method of classification for such patients.
    Jakobovits SL and Travis SPL. Acute Severe Colitis. British Medical Bulletin 2006; 75, 76:131–144.
How well did you know this?
1
Not at all
2
3
4
5
Perfectly
19
Q
  1. A 78-year-old woman is admitted with a dense right-sided weakness. She has reduced consciousness but is moving her left arm and both legs normally. She is confused and disorientated but opens her eyes in response to vocal stimuli and follows commands.
    What is this patient’s score on the Glasgow coma scale?
    A 10
    B 11
    C 12
    D 13
    E 14
A
  1. D 13
    The Glasgow coma scale (GCS) is a 15-point scale for assessing and recording a patient’s conscious state (Table 1.3). It is composed of eye opening, best motor and verbal response.
How well did you know this?
1
Not at all
2
3
4
5
Perfectly
20
Q
  1. ABO compatibility is advisable for the transfusion of blood products. Which of the following products does not require ABO compatibility?
A Cryoprecipitate
B Fresh frozen plasma
C Haemoglobin solution
D Packed red cells
E Platelets
A
  1. C Haemoglobin solution
    Solutions of free haemoglobin do not have the antigenic characteristics of the blood groups, and they therefore do not require compatibility testing. ABO incompatibility can reduce the expected platelet count increment by 10−30%.
    Remy B, Deby-Dupont G, Lamy M. Red blood cell substitutes: fluorocarbon emulsions and haemoglobin solutions. British Medical Bulletin 1999; 55:277–298.
How well did you know this?
1
Not at all
2
3
4
5
Perfectly
21
Q
  1. A 24-year-old woman is involved in a road traffic accident where she sustains a splenic injury.
    Which of the following statements describes the classification of splenic injury?

A Grade I injury describes an intraparenchymal haematoma under 5 cm in diameter
B Grade II injury describes a capsular tear under 1 cm deep
C Grade III injury describes a subcapsular haematoma under 50% of the surface
area
D Grade III injury describes a capsular laceration 1–3 cm deep in the parenchyma
E Grade IV injury describes a laceration producing major devascularisation
(> 25% of spleen)

A
  1. E Grade IV injury describes a laceration producing major devascularisation (> 25% of spleen)
    Grade IV injury describes a laceration producing major devascularisation equivalent to over 25% of the spleen.
    Grade I splenic injuries include subcapsular haematomas (of < 10% of the surface area) and capsular lacerations < 1 cm deep. Grade II splenic injuries include subcapsular haematomas (of < 50% of the surface area) and intraparenchymal haematomas
    < 5 cm. They also include capsular tears between 1 and 3 cm deep. Grade III splenic injuries include expanding subcapsular or intraparenchymal haematomas, ruptured haematomas, and any lacerations > 3 cm (or involving trabecular vessels). Grade IV injuries describe segmental, or hilar, vessel lacerations or major devascularisation. Grade V injuries refer to a completely shattered or devascularised spleen.
    Moore EE, Shackford SR, Pachter HL, et al. Organ injury scaling: spleen and liver. J Trauma 1994; 38:323–324.
How well did you know this?
1
Not at all
2
3
4
5
Perfectly
22
Q
  1. A72-year-oldmanpresentswithfreshrectalbleeding.HeundergoesCTangiography, which fails to localise the bleeding source and continues to bleed.
    What is the most appropriate investigation?
A Barium enema
B CT of chest, abdomen and pelvis
C Colonoscopy
D Flexible sigmoidoscopy
E Upper gastrointestinal endoscopy and proctoscopy
A
  1. E Upper gastrointestinal endoscopy and proctoscopy
    Acute colonic bleeding is a common problem and can manifest in a broad spectrum of clinical signs. The incidence increases with age and in the majority it resolves without intervention. However, in approximately 15% of cases bleeding will continue, resulting in haemodynamic instability. CT angiography has been shown in porcine models to detect bleeding rates of < 0.3 mL/min, but endoscopic visualisation of the upper gastrointestinal tract with endoscopy and proctoscopy should be first line investigations. Haemorrhoids must be excluded as a cause.
    MacKay GJ, Dorrance HR, Molloy RG, O’Dwyer PJ. Colorectal Surgery. Oxford: Oxford University Press, 2010: 393.
    Laing CT, Tobias T, Rosenblum DI, et al. Acute gastrointestinal bleeding: emerging role of multidetector CT angiography and review of current imaging techniques. Radiographics 2007; 27:1055–1070.
How well did you know this?
1
Not at all
2
3
4
5
Perfectly
23
Q
  1. A 55-year-old man presents to the surgical department with acute colonic bleeding.
    Which of the following statements is most appropriate?

A Angiodysplasia is a rare cause
B Diverticular bleeding is the most common cause accounting for approximately
50% of cases
C For angiography to successfully identify the bleeding source, the blood loss
must be above 10 mL/min
D Rectal causes preclude the need for proximal investigation
E The incidence of bleeding post polypectomy is as high as 30%

A
  1. B Diverticular bleeding is the most common cause accounting for approximately 50% of cases
    In the elderly population the commonest cause of massive colonic bleeding is diverticular disease, (up to 50% of cases) closely followed by angiodysplasia (40%) . The incidence of bleeding post polypectomy is < 2%.
    Avots–Avotins KV, Waugh DE. Colonic bleeding in the elderly. Clin Geriatr Med 1985;1(2):433–443.
    MacKay GJ, Dorrance HR, Molloy RG, O’Dwyer PJ. Colorectal Surgery. Oxford: Oxford University Press, 2010: 390.
    Ginsberg G. Risks of Colonoscopy and Polypectomy. Tech Gastrointest Endosc 2008; 10:7–13.
How well did you know this?
1
Not at all
2
3
4
5
Perfectly
24
Q
  1. A 74-year-old woman attends her general practitioner (GP) complaining of absolute constipation and abdominal distension. The GP is concerned about the possibility of large bowel obstruction.
    Which of the following statements describes the features of large bowel obstruction?

A Colonic volvulus is the most common cause
B Colonoscopy is the most appropriate first line investigation
C If the ileocaecal valve is incompetent a closed loop obstruction will develop
D Nausea and vomiting are early clinical symptoms
E Signs of peritonitis suggest ischaemia or perforation

A
  1. E Signs of peritonitis suggest ischaemia or perforation
    Large bowel obstruction is a common surgical emergency. Mechanical causes account for > 60%, colonic tumours account for 20% and volvulus 5%. Nausea and vomiting are late signs as the obstruction is distal in the gastrointestinal tract. CT has largely replaced other investigations as it can distinguish between different pathologies and, if the obstruction is the result of a malignant cause, it will provide information on staging.
    MacKay GJ, Dorrance HR, Molloy RG, O’Dwyer PJ. Colorectal Surgery. Oxford: Oxford University Press, 2010: 393.
How well did you know this?
1
Not at all
2
3
4
5
Perfectly
25
Q
  1. An 88-year-old woman with severe chronic obstructive pulmonary disease presents with abdominal distension. A CT shows an obstructing apple core lesion of the sigmoid colon. She is also noted to have bilobar liver metastases.
    What is the most appropriate first line management?
A Caecostomy
B Colonic stenting
C Hartmann’s procedure
D Sigmoid colectomy
E Subtotal colectomy and end ileostomy
A
  1. B Colonic stenting
    This is a frail patient with significant comorbidities, which would make any surgery very high risk. Even a loop colostomy is associated with significant morbidity and
    a small mortality rate. Stenting of the obstructing colonic lesion can achieve the same functional result as surgical decompression without the need for an operation and defunctioning colostomy. It can be used to allow time for work-up to definitive surgery or in an elderly patient, such as this, can provide good palliation.
    Lamah M, Mathur P, McKeown B, et al The use of rectosigmoid stents in the management of acute large bowel obstruction. J R Coll Surg Edinb 1998; 43:318–321.
How well did you know this?
1
Not at all
2
3
4
5
Perfectly
26
Q
  1. A 72-year-old man with refractory hypotension is commenced on dopamine in the intensive care unit.
    Which of the following statements describes dopamine?

A In high doses it causes peripheral vasodilatation
B In low doses it increases renal blood flow
C It decreases cardiac output
D It decreases splanchnic blood flow
E It is able to cross the blood-brain barrier

A
  1. B In low doses it increases renal blood flow
    Dopamine has varying dose-related effects due to α1-, β1- and dopaminergic activity:
    Low dose effects (at doses < 2 μg/kg/min) are predominantly renal. There is increased renal, cerebral, coronary and mesenteric blood flow with vasodilation because of agonistic action on dopamine receptors in these vascular beds.
    Intermediate dose effects (at doses 2–10 μg/kg/min) are predominantly cardiac. β1-agonist activity results in increased cardiac contractility and heart rate. The increased cardiac output and increased dopaminergic activity results in increased mesenteric perfusion. Slight α1-adrenergic activity may lead to a degree of peripheral vasoconstriction.
    High dose effects (at doses 10–20 μg/kg/min) are vasoconstrictive and cardiac. α1-Adrenergic activity causes marked peripheral vasoconstriction and a rise in blood pressure. There can also be renal and mesenteric vasoconstriction. At very high doses (> 20 μg/kg/min) the overriding adrenergic activity may cause extreme vasoconstriction, which can suppress dopaminergic renal vasodilation and deleteriously affect the renal and peripheral circulation.
    Holmes CL, Walley KR. Bad medicine: low-dose dopamine in the intensive care unit. Chest 2003; 123:1266–1275.
    Dopamine in cardiac failure and shock. BMJ 1977; 17:1563–1564.
How well did you know this?
1
Not at all
2
3
4
5
Perfectly
27
Q
  1. A 22-year-old girl has a suspicious lesion on her right shoulder that requires
    excision biopsy.
    Which of the following statements describes the excision margins for malignant skin lesions?

A In pTis melanoma (in situ) an excision margin of 1 cm is recommended
B In pT1 melanoma (0–1 mm thickness) an excision margin of 2 cm is
recommended
C In pT4 melanoma (> 4 mm thickness) an excision margin of 2 cm is
recommended
D With primary basal cell carcinoma lesions < 2 cm in diameter, an excision
margin of 0.5–1.0 cm is recommended
E With squamous cell carcinoma lesions under 2 cm, a margin of 1 cm is
recommended

A
  1. C In pT4 melanoma (> 4 mm thickness) an excision margin of 2 cm is recommended

For surgical excision of melanomatous lesions:
• In pTis (melanoma in situ) an excision margin of 2–5 mm is recommended
• In pT1 melanoma (0–1 mm thickness) an excision margin of 1 cm is
recommended
• In pT2 melanoma (1–2 mm thickness) an excision margin of 1–2 cm is recommended
• In pT3 melanoma (2–4 mm thickness) an excision margin of 2 cm is recommended
• In pT4 melanoma (> 4 mm thickness) an excision margin of 2 cm is recommended
In patients with squamous cell carcinoma, a 4 mm margin is sufficient to remove microscopic tumor in over 95% of well-differentiated tumours which are < 2 cm
in diameter. A wider margin of 6–10 mm is required for lesions > 2 cm in diameter, less-differentiated, or in high-risk locations (such as the scalp, ears, eyelids, nose, or lips).
In cases of primary basal cell carcinoma (BCC), a 3 mm surgical margin adequately clears the tumour in 85% of patients with lesions under 2 cm. However, increasing the excision margin to 4–5 mm increases the peripheral clearance rate to 95%.
In instances of recurrent BCC, margins of 5–10 mm have been recommended.
Balch CM, Buzaid AC, Soong SJ, et al. Final version of the American Joint Committee on Cancer staging system for cutaneous melanoma. J Clin Oncol 2001; 19:3635–3648.
Brodland DG, Zitelli JA. Surgical margins for excision of primary cutaneous squamous cell carcinoma. J Am Acad Dermatol 1992; 27:241–248.
Burg G, Hirsch RD, Konz B, Braun-Falco O. Histographic surgery: accuracy of visual assessment of the margins of basal-cell epithelioma. J Dermatol Surg Oncol 1975; 1:21–24.
Wolf DJ, Zitelli JA. Surgical margins for basal cell carcinoma. Arch Dermatol 1987; 123:340–344.

How well did you know this?
1
Not at all
2
3
4
5
Perfectly
28
Q
  1. A 60-year-old man presents with large volume haematemesis. After resuscitation an endoscopy reveals bleeding varices in the fundus of the stomach with no other abnormalities seen.
    What is the most appropriate initial treatment?
A Balloon tamponade
B Band ligation
C Blood pressure regulation
D Injection of cyanoacrylate glue
E Transjugular intrahepatic portosystemic shunt
A
  1. D Injection of cyanoacrylate glue
    Unlike oesophageal varices, bleeding from gastric varices is difficult to treat using band ligation because of difficult deployment in a retroflexed scope position. There are few controlled studies of medical therapy for gastric variceal bleeding and most evidence is extrapolated from what is known regarding the management of oesophageal varices.
    The best endoscopic therapy for fundal gastric variceal bleeding is injection of cyanoacrylate glue which hardens on contact with blood. Following healing, when the mucosa sloughs away the overlying glue ‘cap’ will also be shed. Complications include infection and embolism of glue particles (which are fortunately very rare). The glue can also harden inside the scope and this can damage the equipment.
    Feldman M, Friedman LS, Brandt LJ (eds). Sleisenger and Fordtran’s Gastrointestinal and Liver disease, 9th edn. Philadelphia: Saunders, 2010.
How well did you know this?
1
Not at all
2
3
4
5
Perfectly
29
Q
  1. A 66-year-old man has long-standing portal hypertension due to alcohol abuse. He is investigated on an outpatient basis for the development of varices.
    Which of the following anatomical areas does not represent a site of portosystemic varices?

A Intrahepatic, between the portal vein and the inferior vena cava
B Lower oesophagus, between the left gastric vein and oesophageal branches of
the azygous vein
C Rectal, between the superior rectal vein and the pudendal vein
D Retroperitoneal, between the ovarian vessels and the renal veins
E Umbilical, between the obliterated umbilical vein and the left portal vein

A
  1. D Retroperitoneal, between the ovarian vessels and the renal veins
    Retroperitoneal varices occur between the mesenteric vessels and renal (or gonadal, or iliac) veins. However, in patients with infrarenal inferior vena cava obstruction, the ovarian (gonadal) vessels act as a collateral drainage for the systemic circulation by facilitating drainage form the internal iliac vessels to the left renal vein and the cava itself. However, this does not involve a portosystemic anastomosis. Other sites of collateralisation (between the portal and systemic circulations) include:
    • The rectum, where the superior rectal from inferior mesenteric vein anastamoses with the middle and inferior rectal veins/pudendal vein.
    • The paraumbilical region, where the left portal/paraumbilical vein anastomoses with the vestigial umbilical vein/superficial epigastric and caput medusa may result.
    • The distal esophagus, where the left gastric vein anastomoses with oesophageal branches of the azygous vein.
    • Intrahepatic, between the portal vein and the inferior vena cava.
How well did you know this?
1
Not at all
2
3
4
5
Perfectly
30
Q
  1. Which of the following pathogens are not eradicated by alcohol based hand gels?
A Clostridium difficile
B Escherichia coli
C Extended spectrum β-lactamase
D Klebsiella species
E Methicillin-resistant Staphylococcus aureus
A
  1. A Clostridium difficile
    Alcohol-based hand rubs are less effective on soiled hands and are ineffective against
    C. difficile infection. Unfortunately, the hand rub is unable to kill the bacterial spores. World Health Organization (WHO). WHO Guidelines on Hand Hygiene in Health Care: First Global Patient
    Safety Challenge: Clean care is safer care. Geneva: WHO, 2009: 242–245.
How well did you know this?
1
Not at all
2
3
4
5
Perfectly
31
Q
  1. A 47-year-old woman presents to the emergency department with generalised weakness, fatigue and light headedness. Her routine blood gases show: pH 7.34, pO2 9.4 kPa, pCO2 4.4 kPa, HCO3 22 mmol/L. Serum biochemistry includes Na+ 131 mmol/L, K+ 5.1 mmol/L, and plasma glucose 3.4 mmol/L.
What is the most likely diagnosis?
A Addison’s disease
B Chronic renal failure
C Diabetic ketoacidosis
D Exacerbation of chronic obstructive pulmonary disease
E Myasthenia gravis
A
  1. A Addison’s disease
    This patient has biochemical evidence of metabolic acidosis with low bicarbonate, sodium and glucose and an elevated serum potassium. The normal range for serum glucose is 4–7 mmol/L (70–125 mg/dL).
    Addison’s disease (or adrenocortical insufficiency) results in reduced synthesis of mineralocorticoids and glucocorticoids. As a result of decreased glucocorticoid effects, there is a reduction in serum blood glucose. Because of decreased circulating cortisol, the kidney cannot excrete free water and hyponatraemia develops. Metabolic acidosis develops because of low aldosterone levels causing sodium
    loss in the urine, with H+ retention in the serum. Low aldosterone also leads to hyperkalaemia. Clinical symptoms which develop are mainly due to the low blood sugar (hypoglycaemic autonomic effects) and low sodium (hypotension).

Ten S, New M, Maclaren N. Addison’s disease. J Clin Endocrinol and Metabl 2001; 86:2909–2922.

How well did you know this?
1
Not at all
2
3
4
5
Perfectly
32
Q
  1. A 47-year-old man has an open appendicectomy. Subsequently a 2.5 cm carcinoid tumour with clear resection margins is diagnosed on histopathology.
    What is the most appropriate management?
A Chemotherapy
B Colonoscopy
C No further management required
D Right hemicolectomy
E Subtotal colectomy and ileorectal anastomosis
A
  1. D Right hemicolectomy
    The patient requires a right hemicolectomy as the carcinoid of the appendix is > 2 cm in diameter. A right hemicolectomy shoud be considered if the appendix tumour is large (1–2 cm) or invades the serosa. If a right hemicolectomy is to be performed then a regional lymphadenectomy should also be considered.
    Ramage JK, et al. Guidelines for Management of Gastroenteropancreatic Neuroendocrine (including carcinoid) tumours. Gut 2005; 54(Suppl IV): iv1–iv16.
How well did you know this?
1
Not at all
2
3
4
5
Perfectly
33
Q
  1. A study is performed to assess whether there is a correlation between the time a patient waits for colonoscopy (irrespective of outcome) and the overall satisfaction rating for the in-hospital experience. Waiting time is divided into four ascending groups and satisfaction is scored on a continuous rating scale from 1 to 100.
Which statistical test should be used to examine for any association?
A Two-sided t-test
B ANOVA test
C Kruskal–Wallis test
D Linear regression analysis
E Mann–Whitney U test
A
  1. C Kruskall–Wallis test
    When answering a statistical question, it is important to select the correct test (Table 1.4). In this example, a test for an association between two continuous variables is sought. Satisfaction scores (notoriously difficult to attain reliability) are very unlikely to have normal distribution, and so median scores are applicable. Therefore, without assuming equal variances or normal distribution in the data,
    the Kruskal–Wallis test can be applied; where normal distribution and mean values are applied, ANOVA would be the correct test. The Mann–Whitney U test compares medians in two sets of continuous data; the 2-sided t-test compares means and the linear regression analysis is applicable in determining the contribution of multiple factors to one continuous outcome variable.
    Table 1.4 provides a simple overview for comparative statistical tests that should be easily remembered.
    Kirkwood BR, Sterne JAC. Non-parametric methods based on ranks. In: Kirkwood BR, Sterne JAC (eds). Essential Medical Statistics, 2nd edn. Oxford: Wiley-Blackwell, 2004: 345.
How well did you know this?
1
Not at all
2
3
4
5
Perfectly
34
Q
  1. A controlled study is set up to address whether or not patients having single incision laparoscopic cholecystectomy have similar outcomes to standard laparoscopic cholecystectomy. The primary outcome measure is complication rate, estimated at 5% for the standard procedure. To detect a difference of 2.5%, in either direction, it is decided that 200 patients require to be recruited to both arms of the study, with a power of 80%.
    Which statement best describes the application of statistical principles?

A A significant difference (p-value = 0.05) has a likelihood of being due to chance in 1 in 20
B All patients should be offered either procedure by the clinician
C If early results show inferiority, the trial continues to allow conclusions to be
drawn
D If there is low recruitment, there is a risk of a type I error
E The study will be underpowered to detect this difference

A
  1. A A significant difference (p-value = 0.05) has a likelihood of being due to chance in 1 in 20
    A type I (α) error describes the likelihood of wrongly rejecting the null hypothesis. A p-value of 0.05 describes the chance that a seemingly positive finding is actually due to luck alone, and in fact, no real difference exists (0.05 is equivalent to 1 in 20 mathematically).
    It is clear that a power calculation has been carried out, based on the desired power and relevant difference which we wish to detect. The methodology described is
    that of a randomised controlled trial. In specially designed, pragmatic studies,
    either surgeon and/or patient preference may be accounted for, although this is
    not generally the case in controlled trials. Early stopping rules always apply, with patient safety paramount. If there is clearly a higher rate of complication with the new intervention, a trial is not justified, as the whole basis for its carrying one out is that of clinical equipoise and uncertainty. If a trial is underpowered, there may be too few subjects to correctly identify a real difference. This is a false negative or falsely retaining the null hypothesis, and represents a type II (β) error.
    Kirkwood BR, Sterne JAC. Non-parametric methods based on ranks. In: Kirkwood BR, Sterne JAC (eds). Essential Medical Statistics, 2nd edn. Oxford: Wiley-Blackwell, 2004: 425.
How well did you know this?
1
Not at all
2
3
4
5
Perfectly
35
Q
  1. A surgeon would like to know whether patients with thyroid goitres are more or less likely to have been exposed to irradiation during their early years, than those patients without goitres.
    What is the best study design for this purpose?
A Case–control study
B Environmental study
C Longitudinal cohort study
D Meta-analysis of the literature
E Randomised trial of exposure
A
  1. A Case–control study
    Here, we have a condition which we wish to link with a previous exposure. The question may be answered with a population level, longitudinal cohort study, following up children and recording details of any exposure, with rates of goitre later in life, to generate relative risk data. However, this is wholly impractical. It is more straightforward to answer this question by comparing two sets of odds ratios, to attribute the likelihood that someone with a condition has had a certain exposure, compared to the likelihood that someone free of the exposure has the condition. This does not imply causality, but degrees of relationship can be ascertained, and may provide justification for larger scale, longitudinal studies. Thus, the two sets
    of patients have different odds ratios generated. A randomised trial of exposure is simply unethical. An environmental study may act as a hypothesis-generating study during the early stages of studies. A meta-analysis may address the question, but would be unspecific to this surgeon’s own population. It also traditionally pertains only to randomised controlled trials.
    Bhopal R. Concepts of Epidemiology: Integrating the Ideas, Theories, Principles and Methods of Epidemiology, 2nd edn. Oxford: Oxford University Press, 2008.
How well did you know this?
1
Not at all
2
3
4
5
Perfectly
36
Q
  1. A surgeon would like to set up a randomised double-blind controlled trial, examining the efficacy of laparoscopic fundoplication (comparing partial and total wrap). The outcomes are defined and a power calculation performed, and the centre is ready to start recruitment.
    Which factor is not essential to the validity of the trial?

A Adopting a recognised method for patient allocation and stratification
B Blinding the patients
C Blinding the surgical team
D Ensuring it represents a more general population
E Having randomisation conducted through a remote site

A
  1. D Ensuring it represents a more general population
    Although external validity describes the applicability of trial data to the broader population, it is widely acknowledged that participants in randomised controlled trials (RCTs) are not typical patients. These patients self-select (selection bias) as being more interested/motivated around their healthcare issues, and their treatment will be different, regardless of protocols. The Hawthorne effect describes the
    change in behaviour, and potential outcomes, experienced by both participants and clinicians involved in trials, by the very fact they are aware of being studied. The other options simply describe best practice to ensure internal validity in any RCT, and so all must apply for the results to be valid and reliable in themselves.
    McCarney R, Warner J, Iliffe S, et al. The Hawthorne Effect: a randomised, controlled trial. BMC Med Res Methodol. 2007; 7: 30.
How well did you know this?
1
Not at all
2
3
4
5
Perfectly
37
Q
  1. A 95-year-old man is admitted from a nursing home with diarrhoea and is
    subsequently diagnosed with colitis secondary to Clostridium difficile.
    Which of the following statements describes Clostridium difficile infection?

A Cephalosporins are rarely associated with the diagnosis
B First line medical therapy is intravenous metronidazole
C Its presentation is consistently more severe than other forms of gastroenteritis
D Patients with prolonged hospital stays are at particular risk
E Vancomycin is a frequent risk factor

A
  1. D Patients with prolonged hospital stays are at particular risk
    Although vancomycin is rarely associated with Clostridium difficile diarrhoea,
    it is more often used as a second line treatment, after oral metronidazole. Prolonged hospital admissions (and in particular, stays in the intensive care
    unit), are independently associated Clostridium difficile infection. The clinical presentation of this chapter ranges from mild diarrhoea, to severe pancolitis with pseudomembranes, sepsis and perforation. Examples of this are shown in the endoscopic (Figure 1.6) and postoperative (Figure 1.7) images. A low index of clinical suspicion is required for adequate infection control.
    Starr J. Clostridium difficile associated diarrhoea: diagnosis and treatment. BMJ 2005; 331:498–501.
How well did you know this?
1
Not at all
2
3
4
5
Perfectly
38
Q
  1. A 22-year-old girl is admitted with collapse and dehydration. She is suspected of being chronically malnourished.

Which of the following statements describes malnutrition assessment in the acute hospital setting?
A Every malnourished patient should have trace elements checked prior to commencing on supplements
B Formal diagnosis of malnutrition may be made with body mass index (BMI) ≤ 18 kg/m2
C Only patients with clinical or biochemical signs of malnutrition require nutritional screening at admission
D Obese patients do not require nutritional screening
E The Malnutrition Universal Screening Tool calculates risk by patient’s
reduction in BMI.

A
  1. B Formal diagnosis of malnutrition may be made with BMI ≤ 18 kg/m2
    Guidelines state that nutritional screening should be offered to all patients at the point of admission and if a clinical concern arises at any time, then subsequently repeated weekly for inpatients. A routine method to accomplish this is the Malnutrition Universal Screening Tool (MUST) (Table 1.5), based on the patient’s body mass index (BMI), degree of unintentional weight loss, and effect of acute disease. This was developed to ensure a reliable, thorough and reproducible screening technique. Obese patients, as defined by BMI > 30, may still be malnourished in terms of loss
    of lean muscle mass, particularly in the acute illness phase. BMI ≤ 18 (or 20 with associated > 5% weight loss over 3–6 months), or weight loss > 10% in the same period are diagnostic. Patients require baseline trace elements to be checked only prior to commencement of parenteral feeding.
    Elia M. The ‘MUST’ report: Nutritional screeing for adults: a multidisciplinary responsibility. Redditch: Malnutrition Advisory Group of the British Association for Parenteral and Enteral Nutrition (BAPEN), 2003.
    National Institute of Clinical Excellence (NICE). Nutrition support in adults: oral nutrition support, enteral tube feeding and parenteral nutrition (CG32). London: NICE, 2006. Available at: http://www.nice. org.uk/CG32.
How well did you know this?
1
Not at all
2
3
4
5
Perfectly
39
Q
  1. Which of the following statements does not describe evidence based medicine?

A Absolute risk reduction: effect of an intervention minus effect observed in controls
B Number needed to treat: effectiveness of an intervention (equivalent to 1 ÷ absolute risk reduction).
C Odds ratio: likelihood of being exposed to a risk factor
D Prevalence: risk of developing a new condition within a specified period
E Relative risk: likelihood of an outcome, relative to an exposure status

A
  1. D Prevalence: risk of developing a new condition within a specified period
    This describes incidence not prevalence. Odds ratios and relative risks are frequently confused. Odds ratios pertain to retrospective data, working back from an event
    to assess the likelihood of having been exposed to a risk factor. They cannot be used to describe aetiology with certainty, and are useful in hypothesis generation and epidemiological studies. Relative risk ratios are calculated over prospective, longitudinal data, and reliably inform the strength of the relationship between exposure and outcomes. The absolute risk reduction is a numerical calculation, subtracting the risk rate in the control group from that in a study group. This may then be used to calculate the number needed to treat, which indicates the number of patients who require exposure to an intervention in order for any one patient to benefit. This is useful in economic modelling. See Straus et al (2010) for a readable guide to putting these numbers into practice.
    Straus SE, Richardson WS, Glasziou P, et al. Evidence Based Medicine: How to Practice and Teach EBM. 4th edn, London: Churchill Livingstone, 2010.
How well did you know this?
1
Not at all
2
3
4
5
Perfectly
40
Q
  1. Which of the following statements describes nutritional support?

A Enteral nutrition supplements reduce the risk of refeeding syndrome after a period of starvation
B Oral nutritional supplements reduce the risk of infection and length of hospital stay
C Parenteral nutrition is more effective than enteral at reducing infection in malnourished patients
D Parenteral nutrition should be considered in all patients with a significant degree of malnutrition
E Usual nutritional support should include adequate calories (total energy of 25–35 kcal/kg/day)

A
  1. E Usual nutritional support should include adequate calories (total energy 25–35 kcal/kg/day)
    The daily calorie requirement must be adjusted for any acute illness episode, with 0.8–1.5 g protein/kg/day recommended. There is evidence to support the use of oral supplements towards improved outcomes, with absolute risk reduction of infection of 10% in one systematic review. When comparing enteral and parenteral nutrition in the same series, enteral nutrition was superior. However, overall quality of evidence is too poor to make definitive recommendations.
    Parenteral nutrition should only be considered when the gut is:
    • Unusable
    • Insufficient to achieve adequate absorption
    • Inaccessible
    • Non-functional
    • Leaking
    Otherwise, enteral feeding is always the route of choice. It generally offers better outcomes and fewer complications. In the context of starvation or malnutrition, refeeding syndrome may emerge on recommencement of feeding, particularly with enteral supplementation. This is a metabolic disturbance with rapid electrolyte imbalance, driven largely by a change in basal metabolic rate and insulin levels.
    It requires careful biochemical monitoring, and dietary phosphate and vitamin supplementation is usually prescribed to prevent this.
    Koretz RL, Avenell A, Lipman TO, Braunschweig CL, Milne AC. Does enteral nutrition affect clinical outcome? A systematic review of the randomised trials. American Journal of Gastroenterolgy 2007; 102:412–29.
How well did you know this?
1
Not at all
2
3
4
5
Perfectly
41
Q
  1. A 22-year-old boy is seriously injured in a road traffic accident and following discussion with his family, is being considered as an organ donor.
    Which of the following is not a precondition for the diagnosis of brainstem death?

A A PaCO2 of > 7 kPa has been documented
B All underlying metabolic or pharmacological causes have been excluded
C If neuromuscular blockade has been administered, a return to normal state
has been demonstrated
D Tests are performed by two doctors trained in the technique on two separate
occasions
E There is no papillary reflex to light (cranial nerves II and III)

A
  1. A PaCO2 of > 7 kPa has been documented
    The PaCO2 need rise only to 6–6.5 kPa to confirm brainstem death. The other criteria are accurate. In the UK, brainstem death is defined as complete, irreversible loss of brainstem function, made according to well-defined criteria and preconditions, with reversible causes of apnoeic coma excluded, and brainstem areflexia and persistent apnoea confirmed. The cranial nerve reflexes tested are set out in Table 1.6.
    Pallis C, Harley DH. ABC of Brain Stem Death, 2nd edn. London: BMJ Books, 1996.
How well did you know this?
1
Not at all
2
3
4
5
Perfectly
42
Q
  1. Which of the following statements describes the limitations of day case surgery?

A Diabetic patients can easily be managed on a day case list
B Obese patients (BMI >30) should be excluded from day case lists
C Patients must have a responsible carer accompanying them for 24–48 hours
postoperatively
D Patients must have a telephone in the discharge destination
E The procedure should have a low incidence of postoperative complications

A
  1. B Obese patients (BMI >30) should be excluded from day case lists
    Obese patients were originally excluded from day case surgery lists, with BMI limits
    varying considerably between units. A BMI of 30–40 was used as a cut-off, but
    increasingly even super-obese patients are being managed as day cases. It is now
    deemed acceptable to make this judgement on an individual patient basis. The
    other statements correctly describe defined criteria. Full details are available from
    the anaesthesic guidelines for day case surgery.
    Verma R, Alladi R, Jackson I, et al. Day case and short stay surgery: 2. Anaesthesia, 2011; 66:417–434.
How well did you know this?
1
Not at all
2
3
4
5
Perfectly
43
Q
  1. A 3-week-old neonate presents with crying and vomiting and malrotation is suspected.
    Which of the following statements describes malrotation of the gut?

A The caecum fails to descend to a subhepatic position during embryological
development
B It is frequently associated with other congenital abnormalities
C The diagnosis is based on is bilious vomiting in the first few days of life
D There is normal gas pattern on plain X-ray and the diagnosis requires a
contrast study
E With neonatal bilious vomiting, definitive surgery may be deferred until
nutritional issues are addressed

A
  1. A The caecum fails to descend to a subhepatic position
    during embryological development
    Malrotation (Figure 1.2), although commonly presenting in the first few months
    of life, may also present at a later stage, with 0.2% presenting much later in
    adulthood. Presenting in the neonatal period and the first few months of life, it
    constitutes a surgical emergency, and although baseline evaluation of biochemical
    and haematological indices are recommended, they should not delay surgery.
    Radiological investigations typically show a gasless abdomen, with a little stomach
    gas an isolated feature; contrast studies make the definitive diagnosis, normally
    demonstrating a ‘corkscrew’ duodenum with the duodenojejunal flexure lying to the
    right of the midline. It is normally an isolated abnormality and the child will go on
    lead a normal life following surgical correction.
    Youngson GG. Emergency abdominal surgery in infancy and childhood. In: Jones PF, Krukowski ZH,
    Youngson GG (eds). Emergency Abdominal Surgery 3rd edn, Boca Raton: CRC Press, 1998.
How well did you know this?
1
Not at all
2
3
4
5
Perfectly
44
Q
  1. A 4-year-old infant presents with the passage of ‘redcurrant’ stool. The paediatric
    surgeons are suspicious of intussusception.
    Which of the following statements describes intussusception in the paediatric
    population?

A At operation a resection of the affected bowel is always required
B It is caused by a congenital anatomical variant
C It can be treated by air enema without risk
D It most commonly affects the 4- to 6-year-old age group
E It most commonly occurs in an ileocolic segment of bowel

A
  1. E It most commonly occurs in an ileocolic segment
    of bowel
    The most common cause of paediatric intussusception is lymphoid hyperplasia in
    the Peyer’s patches of the gut. The most prominent such tissue occurs along the
    ileocaecal segment (Figure 1.8). Hence, this is not a congenital ‘lead point’ for the
    intussusception as such. Other possible lead points include Meckel’s diverticulum
    and duplication cysts, as well as B lymphoma of the gut. Intussusception most
    commonly presents in the 2 months to 2 years age bracket. A high level of diagnostic
    accuracy is achieved with abdominal ultrasound, which may demonstrate the ‘target’
    sign of bowel within bowel.
    Pneumostatic reduction is routinely attempted, unless the child already has signs
    of perforation/peritonitis, in which case theatre is essential. Prior to pneumatic
    reduction (‘air enema’), intravenous access and preparation for potential theatre
    should be assured. If it fails, operative reduction is often achieved, without the need
    to resect bowel.
    Youngson GG. Emergency abdominal surgery in infancy and childhood. In: Jones PF, Krukowski ZH,
    Youngson GG (eds). Emergency Abdominal Surgery, 3rd edn. Boca Raton: CRC Press, 1998: 82–135.
How well did you know this?
1
Not at all
2
3
4
5
Perfectly
45
Q
  1. A 15-year-old boy is involved in a fight and sustains a stab wound to his right upper quadrant/epigastrium. On presentation to the emergency department he is lucid,
    oxygenating, but complaining of abdominal pain, with a systolic blood pressure of 90mmHg, on his second litre of crystalloid. On examination, he has peritonitis and his blood pressure is not stabilising.
    What is the most appropriate next step in the management plan?

A CT of the abdomen to delineate injuries
B Focused assessment with sonography for trauma scan to confirm fluid/gas in
the peritoneal cavity
C Laparotomy in emergency theatre
D Serum amylase level
E Urinary catheter insertion

A
  1. C Laparotomy in emergency theatre
    This patient requires an urgent laparotomy (the advanced trauma life support
    manual). He is haemodynamically unstable, with evidence of fluid loss (most likely
    blood) and peritonitis, indicating significant intra-abdominal injury. Scans will delay
    definitive management and may be dangerous in this context. No useful information
    will be gleaned from a FAST scan to supplement the clear clinical signs. Delayed
    pancreatitis is associated with blunt trauma to the same region. Catheterisation will
    take place perioperatively, irrespective of the pathology.
    American College of Surgeons. Abdominal Trauma in Advanced Trauma Life Support Manual, 6th edn.
    Chicago: American College of Surgeons, 1997.
How well did you know this?
1
Not at all
2
3
4
5
Perfectly
46
Q
  1. A 4-year-old infant presents with the passage of bloody stool and abdominal pain.
    A provisional diagnosis of Meckel’s diverticulitis is made by a senior paediatrician.

Which of the following statements describes Meckel’s diverticulum?

A It lies 2 feet (60 cm) proximal to the ileocaecal valve
B It can be attached to the umbilicus
C It most commonly presents as an acute abdomen in the paediatric population
D It most commonly presents with peptic ulceration
E It should always be removed if identified incidentally at laparotomy or laparoscopy

A
  1. B It may be attached to the umbilicus
    A Meckel’s diverticulum, the most common congenital abnormality of the
    small intestine, may be attached to the umbilicus. There may be other potential
    connections with a patent vitellointestinal duct with faecal fistula, or partial patency
    of the duct as a sinus to the umbilicus, with potential ongoing secretions. The
    presence of a Meckel’s may be undiagnosed lifelong, but it occurs in approximately
    2% of the population. The three most common presentations include intestinal
    obstruction (40–50%), peptic ulceration (25%) and acute inflammation (20%),
    mimicking appendicitis. There is no indication to remove the lesion if discovered
    incidentally at surgery for another condition.
    Youngson GG. Emergency abdominal surgery in infancy and childhood. In: Jones PF, Krukowski ZH,
    Youngson GG (eds). Emergency Abdominal Surgery, 3rd edn. CRC Press, 1998: 82–135.
How well did you know this?
1
Not at all
2
3
4
5
Perfectly
47
Q
  1. Which of the following statements describes pyloric stenosis?

A Bilious vomiting is frequently associated with the diagnosis
B In over 50% cases, it presents in the first 2 weeks of life
C Infants present with a hypochloremic, hypokalemic metabolic alkalosis
D It most commonly presents in first born female infants
E Surgical correction must take place within the first 24 hours of presentation

A
  1. C Infants present with a hypochloremic, hypokalemic
    metabolic alkalosis
    Infantile hypertrophic pyloric stenosis is a common surgical problem, and often managed by the non-specialist paediatric general surgeon by way of Ramstedt’s pyloromyotomy (Figure 1.9). The precise aetiology remains unknown, but it tends to affect first-born males, some with a family history, at around 4–6 weeks of age. The vomiting is specifically non-bilious in nature. The child usually has been thriving, until insidious onset of vomiting, which then becomes projectile. The speed of presentation for medical help dictates the degree of metabolic disturbance, but the prolonged loss of gastric juices leads to an alkalosis, with potassium also lost in exchange for the kidney’s attempt to preserve chloride. Electrolyte correction, specifically of the hypochloraemia, and volume replacement must be carried out prior to surgical intervention, which may be delayed by a number of days if necessary. Recovery following the pyloromyotomy is normally straightforward.
Youngson GG. Emergency abdominal surgery in infancy and childhood. In: Jones PF, Krukowski ZH,
Youngson GG (eds). Emergency Abdominal Surgery, 3rd edn. CRC Press, 1998: 82–135.
How well did you know this?
1
Not at all
2
3
4
5
Perfectly
48
Q
  1. A 5-year-old child with pain in his right iliac fossa is suspected to have acute
    appendicitis.
    Which of the following statements describes appendicitis in children?

A A child with peritonitis should have volume resuscitation (≥ 20 mL/kg
intravenous fluid) prior to surgery
B Diagnosis is easier to reach than in adults, and tends to present at an earlier
stage
C Elevated white cell count and C-reactive protein are prerequisites before
taking the child to theatre
D Laparoscopic appendicectomy is contraindicated in children under 6 years
of age
E Signs are ordinarily specific, but diagnosis is aided by sequential re-examination

A
  1. A A child with peritonitis should have volume resuscitation
    (≥ 20 mL/kg intravenous fluid) prior to surgery
    This describes the correct fluid resuscitation of any acutely unwell child. Prior to anaesthesia (in the context of sepsis/peritonitis) some volume resuscitation is
    always required. There is no absolute age below which a laparoscopic approach is contraindicated, but its use must depend on the expertise of the surgeon, with
    the small abdominal cavity and stretchy parietal peritoneum posing specific considerations.
    Despite there being fewer differential diagnoses in the paediatric presentation of appendicitis, the behaviour of the disease is different. This is especially true in the
    very young, and often presents at a more advanced stage (Figure 1.10). This perhaps in part reflects difficulties in communication causing a delay in reaching a diagnosis. Active observation, formally reported over 30 years ago, remains the most important diagnostic tool. There is no substitute for repeated reassessment of subjective and objective signs. No one serological or radiological test is entirely reliable. Advanced disease at presentation is not uncommon in the paediatric population – as much as in adult patients, and laparoscopic appendicectomy may be safely performed in the majority of cases.

Jones PF. Active observation in the management of acute abdominal pain in childhood. BMJ 1976;
2:551–553.

49
Q
  1. A 37-year-old male soldier is shot in the shoulder and abdomen and returns to a
    field hospital, where a damage control laparotomy is performed.
    Which of the following statements describes the principles of damage control
    laparotomy?

A Aim to close the abdomen definitively
B In pre-hospital care, utilise abbreviated initial resuscitation and rapid access to
theatre
C Lavage cavities with warmed fluids until normothermia is maintained
D Minimise the number of operative sessions
E Systematically surgically explore to limit bleeding and blood loss

A
  1. B In pre-hospital care, utilise abbreviated initial
    resuscitation and rapid access to theatre

This correctly describes the importance given to ‘source control’ in the damage control laparotomy. The severity of the injury requires this rapid approach, and evidence does support markedly improved survival in trauma centres practicing in this fashion. The approach dictates that the number of returns to theatre does not matter, and, to guard against abdominal compartment syndrome, an open abdomen managed with an appropriate closure device, is the most appropriate outcome from first, or even second surgery. Further, acidotic and coagulopathic patients do not benefit from
prolonged surgical procedures, which would worsen physiological deterioration. The simple aims of stopping gross bleeding and controlling contamination apply. Cavity lavage is not a part of this approach.

Brooks A, Mahoney P, Schwab CW. Critical Care of the Trauma Patient. In: Brooks A, Girling K, Riley B,
Rowlands B (eds). Critical Care for Postgraduate Trainees. London: Hodder Arnold, 2005: 113–121.
Johnson JW, Gracias VH, Schwab CW, et al. Evolution in damage control for exsanguinating penetrating
abdominal injury. Journal of Trauma 2001; 51:261–269.
Shapiro MB, Jenkins DH, Schwab CW, Rotondo MF. Damage Control: collective review. J Trauma 2000;
49:969–978.

50
Q
  1. A 44-year-old woman falls from a significant height sustaining injuries to her head
    and torso.

Which of the following is a component of the secondary survey as per the
advanced trauma life support (ATLS) protocol?
A Alert, voice, pain unresponsive assessment of neurological status
B Control of haemorrhage
C Focused assessment with sonography for trauma scan
D Log roll including checking spinal integrity and anal tone
E Respiratory rate

A
  1. D Log roll including checking spinal integrity and
    anal tone

Within the advance trauma life support (ATLS) protocol, primary survey, and any immediate resuscitative measures work in parallel, and only once each of the five components of the primary survey are ‘stabilised’ may the trauma provider move on to the more detailed secondary survey.
Primary survey includes:
• Airway assessment and maintenance with C-spine control
• Breathing assessment and ventilation if required
• This includes assessment of respiratory rate and symmetry, with inspection,
percussion and auscultation of the lung fields
• Circulation with haemorrhage control
• This is assessed via conscious level, skin color and pulse, with external
haemorrhage identified and controlled
• Disability: rapid assessment of neurological status
• This is assessed via AVPU (Alert /responds to Voice /responds to Pain /
Unresponsive) and pupillary responses
• Exposure/environmental control
• Full exposure of the patient, with re-warming as necessary
ABCDE describes the order of priority, and, if a prior element deteriorates, the
algorithm is repeated in order as required.

Secondary survey is only instigated once
the patient is stabilised and resuscitation is in progress – allowing more time for a
detailed, systematic evaluation, of which the log roll and associated check is part. A focused assessment with sonography for trauma (FAST) scan of the abdomen is an adjunct to the secondary survey, but increasingly being replaced by CT.
Airway and Ventillatory management.

In: Advanced Trauma Life Support manual, 6th edn. Chicago:
American College of Surgeons, 1997.

51
Q
  1. A 19-year-old man presents to resuscitation in the emergency department following a high-energy, two-vehicle head-on road traffic accident. He was
    unrestrained in the front passenger seat. At primary survey, his airway appears patent, but he is alert only to painful stimuli, and his oxygen saturations are falling
    below 92% on re-breathe mask. There is a possibility of pulmonary contusions.
    He is also noted to have significant maxillary swelling and bruising. There is no apparent circulatory compromise.
    What is the next priority in the management plan?

A Airway management – Guedel airway
B Airway management – nasopharyngeal airway
C Airway management – needle cricothyroidotomy
D Airway management – prepare for definitive orotracheal airway and have surgical airway kit ready
E Breathing management – two person bag valve mask

A
  1. D Airway management – prepare for definitive
    orotracheal airway and have surgical airway kit ready

This patient has an emergency need for definitive airway control. He is alert to painful stimuli and therefore he is unsuitable for a guedel airway. A nasopharyngeal airway is relatively contraindicated given the pattern of injuries (as would be a nasotracheal airway). If his airway is compromised by trauma, then a surgical airway
may still be required, but a definitive orotracheal route is preferable with expertise available from anaesthetics. Until the airway is secured, there is little point in
enhancing ventilation, and this is a secondary priority. However, until the airway is secured, every effort should be made to enhance oxygen delivery, and this may be carried out if enough personnel are available. It is not, however, the chief priority.

Airway and Ventillatory management. In: Advanced Trauma Life Support manual, 6th edn. Chicago:
American College of Surgeons, 1997.

52
Q
  1. A 25-year-old female motorcyclist presents to the emergency department having
    sustained blunt trauma to her chest and possible long bone fractures. Her airway
    is patent and secure, but her oxygen saturations remain at 90–92% on re-breathe
    mask. Examination of her chest reveals asymmetric movement, with lesser
    expansion on the left. There are no respiratory sounds, but heart sounds are
    normal. Her systolic blood pressure is 85 mmHg despite 2 L of crystalloid. A chest
    radiograph reveals a ‘white out’ of the left lung field. A trauma chest drain yields
    1600 mL blood, with a further drop in blood pressure.
    What is the most appropriate next step in the management plan?
    A Alert cardiothoracic surgeons urgently
    B Order a CT of the chest
    C Order an echocardiogram
    D Pericardiocentesis
    E Placement of a second, apical chest tube drain
A
  1. A Alert cardiothoracic surgeons urgently

If > 1500 mL is obtained on immediate drainage of a haemothorax, or > 200 mL per hour for 2–4 hours, there is a high likelihood that surgical exploration is required.
There is nothing to indicate haemopericaridum (Beck’s triad consisting of elevated venous pressure, low arterial pressure and muffled heart sounds) so the options to
assess cardiac tamponade are irrelevant. A chest CT may simply waste time because, depending on stability of other parameters, the cardiothoracic surgeons may not
require this and may wish to proceed straight to theatre. Their expertise should be called upon immediately.

Thoracic Trauma. In: American College of Surgeons: Committee on Trauma. Advanced Trauma Life Support
Course: Student Manual, 6th Edn. Chicago: American College of Surgeons, 1997.

53
Q
  1. Which of the following statements describes the features of inotropic agents?

A Adrenaline is frequently the agent of choice in cardiogenic shock
B Dopamine acts primarily on β2-adrenoceptors with positive inotropic and chronotropic effects
C Glyceryl trinitrate is a useful additive agent in cardiac failure to decrease the afterload
D Noradrenaline is frequently the agent of choice in septic shock
E Use of dopexamine may compromise the splanchnic circulation through positive β2-agonism

A
  1. D Noradrenaline is frequently the agent of choice in
    septic shock

Noradrenaline is the most commonly used inotrope in sepsis with hypoperfusion, predominantly acting at α-adrenoceptors to produce vasoconstriction to counter
the hypotension that results from peripheral vasodilatation. The description in (B) applies to dobutamine, which is used in cardiac failure to reduce afterload. Adrenaline stimulates both α- and β-receptors, with respective vasoconstriction and enhanced cardiac contractility. Afterload may be increased, along with myocardial oxygen demands, so that it is not a useful agent in cardiogenic shock. Glyceryl trinitrate is a useful agent in cardiac failure, but through increased availability of nitric oxide, preload is reduced due to vasodilatation and reduced venous return. Dopexamine, active at β2-receptors, aids in splanchnic perfusion, often in the context of cardiac failure. A concise summary of inotropic agents is given in Ashford and Evans (2001).

Ashford R, Evans N. Surgical Critical Care. London: Greenwich Medical Media, 2001.

54
Q
  1. Which of the following physiological abnormalities is not caused by massive
    transfusion?
A Acidaemia
B Coagulopathy
C Hyperkalaemia
D Hyperthermia
E Hypomagnesaemia
A
  1. D Hyperthermia

Massive transfusion is associated with the ‘lethal triad’ of hypothermia, coagulopathy and metabolic acidosis. This is classically described in the rapid deterioration of
severely injured patients, leading to a significant mortality rate when present. Massive transfusion occurs when half of a patient’s circulating volume is transfused
in a period under 24 hours. There is a requirement for close monitoring and optimisation of volume status and tissue oxygenation, as well as control of any bleeding or coagulation abnormalities. There may also be altered levels of ionised calcium or potassium, and the acid–base balance may be affected. There is a tendency for hyperkalaemia, hypokalcaemia and hypomagnesaemia (in part due to the citrate infused).

British Committee for Standards in Haematology, Stainsby D, MacLennan S, et al. Guidelines on the
management of massive blood loss. British Journal of Haematology 2006; 135:634–41.
Gentilello LM, Pierson DJ. Update in non-pulmonary critical care: trauma critical care. American Journal of
Respiratory and Critical Care Medicine 2001; 163:604–7.

55
Q
  1. Which of the following statements describes systemic inflammatory response
    syndrome (SIRS)?

A A heart rate of > 90 beats per minute and a respiratory rate of > 20 breaths per
minute are adequate diagnostic criteria for SIRS
B At least single organ failure is present in SIRS
C Disseminated intravascular coagulation and related bleeding problems are
characteristic of SIRS
D The diagnosis of SIRS requires the presence of a bacteraemia
E Vasoconstriction is the prominent cardiovascular response

A
  1. A A heart rate of > 90 beats per minute and respiratory rate of > 20 breaths per minute are adequate diagnostic
    criteria for SIRS
    Although the definition of systemic inflammatory response syndrome (SIRS)
    is somewhat arbitrary, it represents the beginning of a potentially important
    continuum which may result in multiorgan failure if the process is not abated. SIRS
    is defined by the presence of two or more of the factors listed in Table 1.7. It was
    a working definition designed to help the early diagnosis of critically ill patients,
    derived from a consensus conference of physicians and intensivists in the USA
    around 20 years ago. SIRS is driven by the release and effect of inflammatory mediators such as interleukin-1 and 6, prostaglandins and leukotrienes, as well as catecholamines and insulin. Sepsis is the equivalent clinical state in the presence of bacteraemia. SIRS may occur in response to any systemic insult, such as major surgery, pancreatitis, trauma, etc. Organ dysfunction is frequently evident, but multiorgan dysfunction syndrome, and eventual multiorgan failure can result. Disseminated intravascular coagulation represents a failing haematological system and would not necessarily be seen in SIRS. Vasodilatation is characteristic, with nitric oxide mediated effects.

American College of Chest Physicians/Society of Critical Care Medicine Consensus Conference: definitions for sepsis and organ failure and guidelines for the use of innovative therapies in sepsis. Critical Care
Medicine 1992; 20:864–74.

56
Q
  1. Which of the following statements describes mechanical ventilation?

A It is usually beneficial to acute kidney injury
B It can be used to aid raised intracranial pressure by increasing intrathoracic
pressure
C It must always be instigated when PaO2 > 8 kPa
D Reduced blood pressure and cardiac output is an important side effect
E There is an increase in the patient’s own response to trauma and metabolic
drive

A
  1. D Reduced blood pressure and cardiac output is an important side effect
    Reduced blood pressure and cardiac output is an important side effect of positive pressure ventilation. Increased preload with reduced venous return occurs because of the loss of negative pressure in the intrathoracic pump. Owing to reduced cardiac output, there is reduced renal blood flow, with reduced perfusion, urine output, and potential worsening of any acute injury in the context of acute illness. Although PaCO2 may arbitrarily be considered a reasonable threshold for
    consideration of ventillatory support, the decision to ventilate a patient is individual,
    and in the context of obstructive airways disease, this may be either non-critical, or
    relatively contraindicative, of support. Patients with injuries with raised intracranial pressure may be ventilated at low pressure and high frequency, to drive a reduction in CO2 which will then reduce cerebral blood volume, and consequently pressure. Through ventilation alone, intrathoracic pressure increase will lead to increased intracranial pressures. Through more rapid correction of hypoxia, hypercapnia and acidosis, ventilation decreases the endogenous catecholamine drive on the cardiovascular
    system. This subsequently reduces the impact of the body’s own stress response.

Ashford R, Evans N. Surgical Critical Care. London: Greenwich Medical Media, 2001.

57
Q
  1. A 58-year-old man is a restrained front-seat passenger in a single vehicle road
    traffic accident, with a rapid deceleration into a wall. Although his airway and breathing are satisfactory on primary survey, his blood pressure is labile, and his
    abdomen painful and distended.
    Which combination of injuries is most likely?

A Compression injury with burst liver
B Compression injury with traumatic diaphragmatic hernia
C Deceleration injury with shearing of spleen/kidney
D Deceleration injury with torn ligamentum teres
E Restraint injury with burst injuries to hollow abdominal viscera

A
  1. C Deceleration injury with shearing of spleen/kidney
    This patient is too stable for a significant aortic injury. The mechanism of injury is
    unlikely to produce a compression injury, whereas with correct seatbelt restraint, the
    abdominal organs are less likely to be injured than alternate areas around the girdle of the torso/long bones impacting. More information is available in the appendix of the advanced trauma life suppport (ATLS) manual, regarding mechanics and mechanisms of injury.

Appendix 2: biomechanics of injury. In: Advanced Trauma Life Support manual, 6th edn. Chicago: American College of Surgeons, 1997.

58
Q
  1. A 47-year-old man is brought to the emergency department having been trapped in a burning house for 20 minutes before rescue. He has sustained burns to his
    entire back, and left arm, anteriorly and posteriorly and they are red and painful.
    He does not appear to have airway compromise, and in particular there is no
    singing of nasal hairs. His blood pressure and pulse are acceptable. His estimated
    weight is 80 kg.
    What is the most appropriate resuscitation plan?

A 3 L compound crystalloid over 8 hours, and the same again over the next
16 hours
B 4.5 L compound crystalloid over 8 hours, and the same again over the next
16 hours
C 4.5 L compound crystalloid over 12 hours, and the same again over the next
12 hours
D 8.5 L compound crystalloid over 12 hours
E Bolus resuscitation with colloid until urinary output established

A
  1. B 4.5 L compound crystalloid over 8 hours, and the same again over the next 16 hours
    This patient has sustained 27% body surface, partial thickness burns (Figure 1.11).
    This is a significant volume burn (despite his apparent stability) and he is at risk of
    delayed metabolic sequelae. The goal of management is continued urine production
    around 1 mL/kg/h for the first day and to monitor this all patients require urinary
    catheters. The Parkland formula is used by the ATLS protocol, whereby % burn × kg
    weight × 4 = minimum volume to be infused in first 24 hours postburn. The first half
    of this volume should be given in the first 8 hours and the second half in the next 16
    hours. If the patient appears volume deplete at the outset, these proportions may be varied, and may be titrated against urinary output.

In this patient’s case, he requires: 27% × 80 kg × 4 = 8640 mL total fluid
This equates to 4.32 L in first 8 hours, and the same in the subsequent 16 hours.
The simple charts in Figure 1.11 provide an easy estimate for the % burn sustained
– the higher surface area: volume ratio of children is important.
Hettiaratchy S, Papini R. Initial management of a major burn: II–assessment and resuscitation. BMJ 2004;
329:101–103.

59
Q
  1. The organ failure scoring systems, multiple organ dysfunction score and sequential
    organ failure assessment, have which of the following components in common?

A Creatinine, pulmonary artery wedge pressure, Glasgow coma scale
B Glasgow coma scale, PaO2: FiO2 ratio, serum creatinine
C Platelet count, PaO2: FiO2 ratio, serum/urine osmolarity
D Prothrombin time, PaO2: FiO2 ratio, serum creatinine
E Prothrombin time, serum/urine osmolarity, bilirubin

A
  1. B Glasgow coma scale, PaO2: FiO2 ratio, serum creatinine
    The components of multiple organ dysfunction score (MODS) and sequential organ
    failure assessment score (SOFA) scoring systems are listed below:
MODS
• Respiratory: PaO2: FiO2 ratio
• Renal: serum creatinine
• Hepatic: serum bilirubin
• Cardiovascular: pressure adjusted heart rate, derived from the produce of heart
rate and a pressure ratio
• Haematological: platelet count
• Neurological: Glasgow coma score
SOFA
• Respiratory: PaO2: FiO2 ratio
• Renal: serum creatinine or urine output
• Hepatic: serum bilirubin
• Cardiovascular: hypotension
• Haematological: platelet count

There are similarities between both the components and the uses of the scoring
systems, both being used towards mortality predictions. For example, SOFA > 15 during an intensive therapy unit admission increases predicted mortality to 90%.
For full scoring components and derivative scores see Brooks et al (2005).
Brooks A, Sperry D. Epidemiology of Critical Care. In: Brooks A, Girling K, Riley B, Rowlands B (eds). Critical
Care for Postgraduate Trainees. London: Hodder Arnold, 2005.

60
Q
  1. A 72-year-old man with a previous history of deep vein thrombosis undergoes a straightforward anterior resection with primary anastomosis for a high rectal
    carcinoma. At postoperative day 4, he appears ready to go home, but complains of a swollen, tender right leg. He is diagnosed with a deep vein thrombosis, commencing therapeutic dose low-molecular weight heparin immediately, with a plan for warfarinisation. That night, he becomes acutely unwell, with dyspnoea, abdominal pain and distension, and high fever. Free gas is seen on his chest X-ray. Following instigation of intravenous fluids and antibiotics, what is the most appropriate next step in the management plan?
    A Conservative management of anastomotic leak with further fluids, antibiotics, +/– radiological drain and commence warfarinisation
    B Conservative management of anastomotic leak with further fluids, antibiotics,
    +/– radiological drain and continue to administer heparin
    C Placement of a caval filter prior to return to theatre for anastomotic leak
    D Return to theatre for anastomotic leak
    E Reversal of heparin prior to return to theatre for anastomotic leak
A
  1. C Placement of caval filter, then definitive surgical
    treatment of the leak
    This man has systemic compromise from his anastomotic leak. This will not settle with
    conservative management, and he therefore needs to return to theatre for control of
    sepsis and defunctioning. To do this safely, in the presence of a venous thrombosis,
    further prophylaxis against embolism is required. Emergency placement of a caval
    filter is also required. Low-molecular-weight heparin cannot be reversed, but this
    should neither preclude surgical intervention for his sepsis, nor filter placement. The
    evidence for filters is relatively weak but is outlined in a recent Cochrane summary.
    Young T, Tang H, Hughes R. Vena caval filters for the prevention of pulmonary embolism. Cochrane
    Summaries, 2010.
61
Q
  1. A 63-year-old diabetic man is admitted as an emergency to the surgical ward and
    diagnosed with idiopathic, acute severe pancreatitis. He rapidly deteriorates, with
    a need for ventilatory support, and his oliguria evolves into anuria within 48 hours.
    He is obese, but his abdomen is distended and tender, with apparent diaphragmatic
    splinting. His arterial gas pattern shows a mild metabolic acidaemia, with a PaO2 of
    7.8 kPa and PaCO2 of 6.0 kPa on 80% FiO2. Non-invasive ventilation is considered,
    and he requires renal haemofiltration.
    What is the most likely diagnosis?
A Abdominal compartment syndrome
B Acute kidney injury
C Established renal failure
D Necrotic pancreatitis
E Type 2 respiratory failure
A
  1. A Abdominal compartment syndrome
    The inflammatory response and intra-abdominal swelling which occurs secondary
    to acute severe pancreatitis is driving the pathophysiology in this scenario.
    Abdominal compartment pressure may be estimated by elevation of a urinary
    catheter from the level of the pubis. A pressure above 12 cm is elevated and
    > 20 cm is diagnostic of abdominal compartment syndrome (ACS). By reducing
    abdominal blood flow and diaphragmatic movement, the secondary compromise
    in renal and respiratory function are worsened, and pose particular management
    problems in terms of ventilating the patient against this increased pressure. The
    renal impairment remains acute, but is more complex than a straightforward acute
    renal injury. The form of pancreatitis is immaterial to the pathological outcome
    here. This phenomenon is increasingly recognised, and actively managed, in the
    acute surgical patient, often using a vacuum assisted closure device (Figure 1.12).

Anderson IA. Abdominal sepsis and abdominal compartment syndrome. In: Paterson-Brown S (ed),
Core Topics in General and Emergency Surgery: A Companion to Specialist Surgical Practice, 4th edn.
Philadelphia: Saunders Elsevier, 2009: 331–343.

62
Q
  1. A 45-year-old woman with Crohn’s disease develops sudden onset bilious
    vomiting and abdominal distension. Her history includes a previous right
    hemicolectomy for terminal ileal disease 15 years ago with two further small bowel
    resections.
A Adhesions
B Bezoar
C Caecal carcinoma
D Diaphragmatic hernia
E Femoral hernia
F Gallstone ileus
G Incisional hernia
H Inguinal hernia
I Postoperative ileus
J Pseudomyxoma peritonei
K Recurrent Crohn’s disease
L Small bowel tumour
A
  1. A Adhesions
    The history of Crohns’ disease is slightly misleading as there are no features here
    to suggest a ‘flare-up’ or recurrent disease. A more protracted history of pain or
    discomfort with increased bowel frequency and blood or mucus in the motions
    would be more classical and suggestive of active disease. Here the relevant history
    is of multiple laparotomies which will predispose the patient to the development
    of adhesions. Similar questions may push the candidate for a suitable management
    plan in the above clinical scenario. Most texts agree on a period (normally < 24 hours) of conservative management, including resting of the gastrointestinal tract, fluid resuscitation (with or without nasogastric aspiration or drainage if large volume or repeated vomits feature).
63
Q
  1. A 78-year-old woman describes recent vague upper abdominal pain and now
    has profuse vomiting. Erect chest X-ray demonstrates an unusual gas pattern in
    the right upper quadrant and abdominal X-ray shows dilated small bowel loops
    proximal to an opacification in the right lower quadrant.
A Adhesions
B Bezoar
C Caecal carcinoma
D Diaphragmatic hernia
E Femoral hernia
F Gallstone ileus
G Incisional hernia
H Inguinal hernia
I Postoperative ileus
J Pseudomyxoma peritonei
K Recurrent Crohn’s disease
L Small bowel tumour
A
  1. F Gallstone ileus
    In patients aged over 65 years, gallstone ileus makes up approximately 25% of non-strangulated small bowel obstruction and carries a high mortality
    rate (approximately 15%). The unusual gas pattern seen here probably refers
    to pneumobilia (or gas in the biliary tree), and the opacification in the right
    lower quadrant is likely to represent a gallstone lodged in the terminal ileum.
    In conjunction with dilated fluid-filled loops of small bowel, these are classical
    radiographic findings associated with gallstone ileus. Despite this, the sensitivity of a plain radiograph for diagnosis of gallstone ileus varies from 40–70%.This compares to CT, which has a sensitivity and diagnostic accuracy of close to 100%.

Maglinte DD, Reyes BL, Harmon BH, et al. Reliability and role of plain film radiography and CT in the diagnosis of small-bowel obstruction. Am J Roentgol 1996; 167:1451–1455. Reisner RM, Cohen JR. Gallstone ileus: A review of 1001 reported cases. Am J Surg 1994; 60:441–446. Rigler LG, Borman C, Noble JF. Gallstone obstruction: Pathogenesis and roentgen manifestations. JAMA 1941; 117:1753–1759.

64
Q
  1. A 35-year-old man with cachexia, lower abdominal pain and vomiting undergoes
    laparoscopy for planned appendicectomy. After encountering difficulties creating
    a pneumoperitoneum there is very poor visibility because of gelatinous material in
    the peritoneal cavity.
A Adhesions
B Bezoar
C Caecal carcinoma
D Diaphragmatic hernia
E Femoral hernia
F Gallstone ileus
G Incisional hernia
H Inguinal hernia
I Postoperative ileus
J Pseudomyxoma peritonei
K Recurrent Crohn’s disease
L Small bowel tumour
A
  1. J Pseudomyxoma peritonei
    Pseudomyxoma peritonei (PMP) is a rare clinical entity (affecting approximately
    one individual per million population). It is a form of malignancy characterised by mucin production which can progress to fill the abdomen and compress hollow
    organs. When picked up intraoperatively, the findings are surprising as the volume
    of mucinous material can be quite marked. Some people develop symptoms as a
    result of direct compression but PMP is considered only borderline malignant due
    to its slow clinical progression. It is associated with appendixeal and mucinous
    ovarian tumours and should always be suspected in patients with these diagnoses.
    Treatment is usually based on minimising symptoms and ‘watchful waiting’, unless
    progression is marked or the symptoms dictate intervention. In such cases, the Sugarbaker procedure (peritonectomy) is used for debulking with postoperative
    chemotherapy (often intraperitoneally).
    National Institute for Health and Clinical Excellence (NICE).Complete cytoreduction for pseudomyxoma
    peritonei (Sugarbaker technique) IPG56. London: NICE, 2004. http://www.nice.org.uk/nicemedia/
    live/11060/30806/30806.pdf
65
Q
  1. A test for comparison of proportions.
A ANOVA
B χ2 test
C Friedman’s test
D Independent t-test
E Kruskal–Wallis
F Kaplan–Meier
G Mann–Whitney U test
H Multivariate analysis
I Paired t-test
J Pearson’s correlation coefficient
K Spearman’s rank correlation
L Wilcoxon signed-rank testing
A
  1. B χ2 test
    When addressing a research question, it is important to differentiate between
    ‘parametric’ and ‘non-parametric’ data before selecting the appropriate method
    of description and the most accurate statistical test (Table 1.8). ‘Parametric’ refers
    to data which is normally distributed and follows Gaussian distribution, unlike
    ‘non-parametric’ data which is positively or negatively skewed. For practical
    purposes this can be determined by visualising the distribution of data on a
    histogram using a statistical package.

Parametric tests of normally distributed data include (Student’s) paired t-test and
independent t-test, which assess the distribution and central location of data
between paired and unpaired cases respectively.
χ2 testing is a comparison of proportions and is independent of whether data is
normally distributed or not. However, it is restricted in its accuracy when values are
low, and in such cases a Fisher’s exact test must be used.

66
Q
  1. A non-parametric test for comparison of central location between two
    independent samples.
A ANOVA
B χ2 test
C Friedman’s test
D Independent t-test
E Kruskal–Wallis
F Kaplan–Meier
G Mann–Whitney U test
H Multivariate analysis
I Paired t-test
J Pearson’s correlation coefficient
K Spearman’s rank correlation
L Wilcoxon signed-rank testing
A
  1. G Mann–Whitney U test
    The Mann–Whitney U test is a non-parametric statistical test used to assess whether
    two independent samples of observations have differing values. Contrast this with the Wilcoxon signed-rank test, which is a non-parametric statistical test for two
    related samples or repeat observations on a single sample.
67
Q
  1. A non-parametric test for evaluating the difference between several related
    samples.
A ANOVA
B χ2 test
C Friedman’s test
D Independent t-test
E Kruskal–Wallis
F Kaplan–Meier
G Mann–Whitney U test
H Multivariate analysis
I Paired t-test
J Pearson’s correlation coefficient
K Spearman’s rank correlation
L Wilcoxon signed-rank testing
A
  1. C Friedman’s test
    Friedman’s test is a non-parametric test also used to detect differences between
    several related samples, or in one sample across repeated time points.
    Kirkwood B, Sterne J. Essential medical statistics. Oxford: Wiley-Blackwell, 2003
68
Q
  1. A 48-year-old woman has an uneventful laparoscopic cholecystectomy for
    symptomatic gallstones. Six days following surgery she represents complaining of
    upper abdominal pain and distension
A Abdominal radiograph
B Abdominal ultrasound
C Arterial blood gases
D Barium enema
E Blood cultures
F Chest radiograph
G CT of the chest
H CT of the abdomen
I Gastrografin enema
J Laparoscopy
K Laparotomy
L Urine culture
M Wound swab
A
  1. B Abdominal ultrasound
    Following laparoscopic cholecystectomy, complications are relatively common and
    imaging studies may be required to assess for the presence of a fluid collection
    (which may represent abscesses, bilomas, or haematomas). These will not always
    require intervention as a small collection is commonly seen in the postoperative
    period. The easiest way to assess non-invasively is through abdominal ultrasound,
    which will provide information on the presence and amount of any collection and
    can also describe its homogeneity. As a first line investigation it is also useful for
    identifying a retained stone or any biliary dilation.
    Thurley PD, Dhingsa R. Laparoscopic cholecystectomy: postoperative imaging. AJR 2008; 191(3): 794–801.
    Lohan D, Walsh S, McLoughlin R, Murphy J. Imaging of the complications of laparoscopic cholecystectomy.
    Eur Radiol 2005; 15: 904 –912.
    McAlister VC. Abdominal fluid collection after laparoscopic cholecystectomy. Br J Surg 2000; 87: 1126–1127.
69
Q
  1. A 78-year-old man undergoes a Lichtenstein repair of a painful left inguinal hernia. Three weeks later, he complains of gradual onset left lower quadrant pain
    and tenderness. He has a past medical history of chronic obstructive pulmonary disease, diverticular disease and type 2 diabetes.
A Abdominal radiograph
B Abdominal ultrasound
C Arterial blood gases
D Barium enema
E Blood cultures
F Chest radiograph
G CT of the chest
H CT of the abdomen
I Gastrografin enema
J Laparoscopy
K Laparotomy
L Urine culture
M Wound swab
A
  1. H CT of the abdomen
    Although this patient has had a recent hernia operation, the medical history of
    diverticular disease cannot be ignored, particularly in view of the tenderness. A CT
    of the abdomen with intravenous and oral contrast should be performed to reject
    or confirm a diagnosis of diverticulitis. Pericolic fat infiltration is a diagnostic feature.
    Chronic pain following hernia repair does not tend to occur as high as the iliac fossa
    or lower quadrant, unless there is large or small bowel involvement.
    Simmang CL, Shires GT. Diverticular disease of the colon. In: Feldman M, Friedman LS, Sleisenger MH (eds).
    Sleisenger and Fordtran’s gastrointestinal and liver disease: pathophysiology, diagnosis, management, 7th
    edn. Philadelphia: Saunders, 2002: 2100–2112.
    Shen SH, Chen JD, Tiu CM, et al. Colonic diverticulitis diagnosed by computed tomography in the ED. Am J
    Emerg Med 2002; 20:551–557.
70
Q
  1. An 81-year-old man undergoes an anterior resection with primary anastomosis
    for a T3 N1 carcinoma of his mid-rectum. On the sixth postoperative day he has an
    ongoing ileus with diffuse lower abdominal pain. He has a mild fever at 37.6°
    C and abdominal examination reveals fullness in the left iliac fossa which is tender. There
    is a leukocytosis of 15 × 1000 per mm3
A Abdominal radiograph
B Abdominal ultrasound
C Arterial blood gases
D Barium enema
E Blood cultures
F Chest radiograph
G CT of the chest
H CT of the abdomen
I Gastrografin enema
J Laparoscopy
K Laparotomy
L Urine culture
M Wound swab
.
A
  1. H CT of the abdomen
    If an anastomotic leak (and possible collection or abscess) is suspected, a CT of the
    abdomen and pelvis with intravenous, oral, and rectal contrast is indicated. This is
    a highly sensitive (95%) test for identifying abdominal or pelvic abscesses. It also
    affords the opportunity for CT-guided drainage, which is successful in up to 85% of
    patients. Gastrografin enema is another commonly used technique which will provide
    information on the presence of a leak, but will not demonstrate if there is a drainable
    collection and further imaging will be required. This patient has evidence of
    sepsis and the tenderness suggests there may be a collection or abscess. This will
    require drainage and, depending on the extent of the leak, and the integrity of the
    anastomosis, a laparotomy for drainage or defunctioning may be required.
    DuBrow RA, David CL, Curley SA. Anastomotic leaks after low anterior resection for rectal carcinoma:
    evaluation with CT and barium enema. Am J Roent 1995; 165:567–571.
    Nasirkhan MU, Abir F, Longo W, Kozol R. Anastomotic disruption after large bowel resection. World J
    Gastroenterol 2006; 12:2497–2504.
    Lucey BC, Maher MM, Boland GW, et al. percutaneous treatment by interventional radiologists of
    anastomotic leaks: basic concepts. Am J Roent 2002; 179:365–369.
71
Q
  1. An 85-year-old man undergoes open surgical repair of a symptomatic hernia.
    During the operation the hernial sac is found to contain an inflamed appendix.
A Amyand’s
B Diaphragmatic
C Femoral
D Incisional
E Lanz
F Littre’s
G Maydl’s
H Pantaloon
I Paraumbilical
J Petit’s
K Richter’s
L Spigelian
M Umbilical
A
  1. A Amyand’s
    Amyand is reported to have performed the first appendicectomy in 1735, on a
    patient with a perforated appendix situated in an inguinal hernia. This condition is
    very rare with an appendix in the sac of an inguinal hernia reported in approximately
    1% of patients when the appendix is non-inflamed (and 0.08% when inflamed).
    Amyand C. Of an inguinal rupture, with a pin in the appendix coeci, incrusted with stone; and some
    observations on wounds in the guts. Phil Trans Royal Soc 1736; 39:329–342.
    D’Alia C, Lo Schiavo MG, Tonante A, et al. Amyand’s hernia: case report and review of the literature. Hernia
    2003; 7:89–91.
72
Q
  1. A 45-year-old man has two adjacent loops of small bowel that are located within
    a hernial sac with a tight neck. The portion of bowel in-between is deprived of its
    blood supply and is progressing towards necrosis.
A Amyand’s
B Diaphragmatic
C Femoral
D Incisional
E Lanz
F Littre’s
G Maydl’s
H Pantaloon
I Paraumbilical
J Petit’s
K Richter’s
L Spigelian
M Umbilical
A
  1. G Maydl’s
    A Maydl’s hernia occurs when a ‘W-shaped’ segment of small bowel occupies
    the hernia sac. The middle part is situated inside the abdomen and may become
    strangulated, as can also happen to the adjacent loops of small bowel (Figure 1.13).
    Ganesaratnam M. Maydl’s hernia: report of a series of seven cases and review of the literature. Br J Surg
    1985; 72:737–738
73
Q
  1. A 70-year-old obese woman has a right-sided swelling due to a combination of
    direct and indirect hernia. At operation the hernial sacs bulge on either side of the
    inferior epigastric vessels.
A Amyand’s
B Diaphragmatic
C Femoral
D Incisional
E Lanz
F Littre’s
G Maydl’s
H Pantaloon
I Paraumbilical
J Petit’s
K Richter’s
L Spigelian
M Umbilical
A
  1. H Pantaloon
    A Pantaloon hernia is a combination of direct and indirect herniae. The hernial sacs
    bulge on either side of the inferior epigastric vessels like ‘pantaloons’.
    Mirilas P, Mouravas Y. Enigmatic images: inguinal hernia of a ‘third kind,’ pantaloon hernia, ‘direct
    pantaloon’ hernia, or direct hernia and supravesical hernia? Hernia 2010; 14:333–334.
74
Q
  1. A baseline diagnostic test to differentiate between ongoing or resolved hepatitis B
    in unwell patients. This test is used to determine the transmissibility of hepatitis B.
A Hepatitis C virus (HVC) polymerase
chain reaction (PCR)
B Hepatitis B core antibody
C Hepatitis B DNA
D Hepatitis B e antibody
E Hepatitis B e antigen
F Hepatitis B IgM core antibody
G Hepatitis B surface antibody
H Hepatitis B surface antigen
I Hepatitis C antibody
J HIV type 1 serology
K HIV type 2 serology
L Human T-lymphotropic virus serology
A
  1. H Hepatitis B surface antigen
    Hepatitis B surface antigen (HBsAg) is a baseline diagnostic test to differentiate
    between ongoing or resolved hepatitis B in unwell patients. This test is used to
    determine the transmissibility of hepatitis B.
    Inaba N, Ohkawa R, Matsuura A, et al. Sexual transmission of hepatitis B surface antigen. Infection of
    husbands by HBsAg carrier-state wives. Br J Vener Dis 1979; 55(5): 366–368.
    Yuen MF, Wong DK, Lee CK, et al. Transmissibility of hepatitis B virus (HBV) infection through blood
    transfusion from blood donors with occult HBV infection. Clin Infect Dis 2011; 52:624–632.
75
Q
  1. A test to assess response to immunisation in healthcare workers.
A Hepatitis C virus (HVC) polymerase
chain reaction (PCR)
B Hepatitis B core antibody
C Hepatitis B DNA
D Hepatitis B e antibody
E Hepatitis B e antigen
F Hepatitis B IgM core antibody
G Hepatitis B surface antibody
H Hepatitis B surface antigen
I Hepatitis C antibody
J HIV type 1 serology
K HIV type 2 serology
L Human T-lymphotropic virus serology
A
  1. G Hepatitis B surface antibody
    Hepatitis B surface antibody (anti-HBsAg) is a test to assess response to immunisation
    in healthcare workers. Such ‘at-risk’ groups of people receive three vaccinations of
    the the hepatitis B surface antigen (HBsAg) at 1 and 6 months intervals. The hepatitis
    B surface antibody usually becomes measurable in the serum about 4 weeks after
    HBsAg is no longer measurable.
    When antibody is present, it suggests that the infection is no longer active and the
    subject is immune to hepatitis B infection. It is therefore used to determine the need
    for further vaccination.
    Pasko MT, Beam TR. Persistence of anti-HBs among healthcare personnel immunized with hepatitis B
    vaccine. Am J Public Health 1990; 80:590–593.
76
Q
  1. A test used by specialists to monitor hepatitis C treatment.
A Hepatitis C virus (HVC) polymerase
chain reaction (PCR)
B Hepatitis B core antibody
C Hepatitis B DNA
D Hepatitis B e antibody
E Hepatitis B e antigen
F Hepatitis B IgM core antibody
G Hepatitis B surface antibody
H Hepatitis B surface antigen
I Hepatitis C antibody
J HIV type 1 serology
K HIV type 2 serology
L Human T-lymphotropic virus serology
A
  1. A Hepatitis C virus (HVC) polymerase chain reaction (PCR)
    HCV PCR is a test used by hepatologists and virologists to monitor hepatitis C
    treatment by measuring the expression of HCV RNA.
    Urdea MS, Wuestehube LJ, Laurenson PM, Wilber JC. Hepatitis C diagnosis and monitoring. Clinical
    Chemistry 1997; 43:1507–1511.
77
Q
  1. A 58-year-old woman represents with recurrent upper abdominal pain. She has
    had a recent abdominal ultrasound which demonstrated stones in the gallbladder
    with no ductal abnormality. She is apyrexial with normal inflammatory markers,
    but her liver function tests are grossly abnormal.
A CT
B Diagnostic endoscopic retrograde
cholangiopancreatography (ERCP)
C Endoscopic ultrasound
D ERCP with balloon trawl with or
without sphincterotomy
E Fine-needle aspiration
F Laparoscopic cholecystectomy alone
G Laparoscopic cholecystectomy with
on-table cholangiogram
H Laparoscopic cholecystectomy with
bile duct exploration
I Lithotripsy
J Magnetic resonance
cholangiopancreatography (MRCP)
K Percutaneous transhepatic
cholangiography (PTC)
L Ursodeoxycholic acid
M Ultrasound
A
  1. J Magnetic resonance cholangiopancreatography
    (MRCP)
    This patient has recurrent upper abdominal pain, with a recent ultrasound showing
    stones in the gallbladder. She is apyrexial with normal inflammatory markers and is
    therefore unlikely to have cholecystitis or cholangitis. With grossly abnormal liver
    function test values, she is likely to have ductal stones. There seems little value in
    repeating an ultrasound scan as the only difference to management would be if it
    clearly demonstrated ductal stones with dilatation requiring decompression.

MRCP utilises magnetic resonance imaging to examine the biliary and pancreatic
tree in great detail, although it does not offer any therapeutic options. Compared
to an invasive procedure like ERCP, there is very minimal mortality or morbidity
with MRCP. Performing this procedure will provide information on the presence of
any ductal stones and will therefore guide the need for further treatment. A major
limiting factor in patients undergoing MRCP is that some will require a subsequent
ERCP regardless.

Kaltenthaler E, Vergel YB, Chilcott J, et al. A systematic review and economic evaluation of
magnetic resonance cholangiopancreatography compared with diagnostic endoscopic retrograde
cholangiopancreatography. Health Technol Assess 2004; 8:iii, 1–89.

78
Q
  1. A 44-year-old woman presents with constant right upper quadrant pain and
    pyrexia. An ultrasound scan shows a thickened gallbladder containing sludge and
    a common bile duct (CBD) diameter of 15mm. No ductal stones are identified but
    there are poor views of the distal CBD.
A CT
B Diagnostic endoscopic retrograde
cholangiopancreatography (ERCP)
C Endoscopic ultrasound
D ERCP with balloon trawl with or
without sphincterotomy
E Fine-needle aspiration
F Laparoscopic cholecystectomy alone
G Laparoscopic cholecystectomy with
on-table cholangiogram
H Laparoscopic cholecystectomy with
bile duct exploration
I Lithotripsy
J Magnetic resonance
cholangiopancreatography (MRCP)
K Percutaneous transhepatic
cholangiography (PTC)
L Ursodeoxycholic acid
M Ultrasound
A
  1. D ERCP with balloon trawl with or without
    sphincterotomy
    This woman presents with acute cholecystitis and a dilated common bite duct (CBD)
    (based on her ultrasound findings). She will require a cholecystectomy at some
    point, but her biliary tree needs decompressed and this should be performed with
    endoscopic retrograde cholangiopancreatography (ERCP)

This patient is high risk as her CBD is dilated beyond 10mm. As with other high-risk
patients (with cholangitis or pancreatitis) she should undergo ERCP +/– duct
decompression. Up to a third of such patients will have no evidence of a gallstone at
ERCP and further tests may be required at a later date.
Mesenas SJ. Does the advent of endoscopic ultrasound sound the death knell for endoscopic retrograde
cholangiopancreatography (ERCP)? Ann Acad Med Singapore 2006; 35:89–95.

79
Q
  1. A 64-year-old man is admitted as an emergency with minor severity gallstone
    pancreatitis. An abdominal ultrasound scan confirms pancreatitis with gallstones
    in the gallbladder. There is no duct dilatation.
A CT
B Diagnostic endoscopic retrograde
cholangiopancreatography (ERCP)
C Endoscopic ultrasound
D ERCP with balloon trawl with or
without sphincterotomy
E Fine-needle aspiration
F Laparoscopic cholecystectomy alone
G Laparoscopic cholecystectomy with
on-table cholangiogram
H Laparoscopic cholecystectomy with
bile duct exploration
I Lithotripsy
J Magnetic resonance
cholangiopancreatography (MRCP)
K Percutaneous transhepatic
cholangiography (PTC)
L Ursodeoxycholic acid
M Ultrasound
A
  1. G Laparoscopic cholecystectomy with on-table cholangiogram
    This patient is at low risk of gallstone-related complications and should proceed to
    cholecystectomy without further preoperative diagnostic procedures. He should
    also have an intraoperative cholangiogram (IOC) to demonstrate any distal common
    bite duct (CBD) or pancreatic duct stones.
80
Q
  1. A 29-year-old woman complains of perianal pain. She describes occasional
    bleeding and discharge from a midline opening which is 6 cm posterior to the anal
    verge in natal cleft.
A Anal carcinoma
B Anal fissure
C Fistula-in-ano
D Ischiorectal abscess
E Perianal abscess
F Perianal Crohn’s
G Perianal haematoma
H Pilonidal sinus
I Proctalgia fugax
J Proctitis
K Prolapsed thrombosed haemorrhoid
L Rectal carcinoma
M Solitary rectal ulcer syndrome
N Tuberculosis
A
  1. H Pilonidal sinus
    Pilonidal disease can be asymptomatic, or may present with an abscess or discharging
    sinus tract (most commonly in the natal cleft region). It occurs due to ingrowing hairs
    and is more common in male, hirsute patients who sit for prolonged periods.
    Søndenaa K, Andersen E, Nesvik I, Søreide JA. Patient characteristics and symptoms in chronic pilonidal
    sinus disease. Int J Colorectal Dis 1995; 10:39–42.
81
Q
  1. A 54-year-old man has pain after defaecation and has also noticed blood on the
    toilet paper.
A Anal carcinoma
B Anal fissure
C Fistula-in-ano
D Ischiorectal abscess
E Perianal abscess
F Perianal Crohn’s
G Perianal haematoma
H Pilonidal sinus
I Proctalgia fugax
J Proctitis
K Prolapsed thrombosed haemorrhoid
L Rectal carcinoma
M Solitary rectal ulcer syndrome
N Tuberculosis
A
  1. B Anal fissure
    An anal fissure is a small tear or ulcer at the anal verge (normally in the 6 o’clock
    position). It has been suggested it is due to ischaemia with impaired healing due to a
    combination of increased sphincter tone, fibrotic scarring and repeated mechanical
    injury. When an anal fissure develops, it may cause severe sharp pain on defaecation
    which can last for hours afterwards. There may also be fresh blood on the toilet
    paper or in the pan after defaecation. A sentinel pile (a skin tag distinct from a
    haemorrhoid) is a common marker of an anal fissure and also prognosticates that it
    may be difficult to manage conservatively.
    Ayantunde AA, Debrah SA. Current concepts in anal fissures. World J Surg 2006; 30:2246–2260
82
Q
  1. A 44-year-old woman has noticed the discharge of foul smelling fluid from lateral
    to the anal verge. She has a visible pit in the midline.
A Anal carcinoma
B Anal fissure
C Fistula-in-ano
D Ischiorectal abscess
E Perianal abscess
F Perianal Crohn’s
G Perianal haematoma
H Pilonidal sinus
I Proctalgia fugax
J Proctitis
K Prolapsed thrombosed haemorrhoid
L Rectal carcinoma
M Solitary rectal ulcer syndrome
N Tuberculosis
A
  1. C Fistula-in-ano
    The discharge of fluid in this patient’s case represents a fistula. Fistula-in-ano is a feature of the chronic stage of abscess formation and can occur due to simple cryptoglandular
    infection or trauma, Crohn’s disease tuberculosis, cancer or radiation. The external
    opening can be used to locate the internal opening by following Goodsall’s rule.
    Approximately 90% of fistulae with an external opening anterior to the midline will open
    into the anal canal anteriorly (in a radial fashion). Fistulous openings posterior to the
    midline will follow a horseshoe shape to open posteriorly in the midline (Figure 1.15).
    Goodsall DH. Anorectal Fistula. In: Goodsall DH, Miles WE. Diseases of the anus and rectum, part I. London:
    Longmans, Green, 1900:92.
83
Q
  1. A 32-year-old man is struck in the groin by a football and on self-examining he
    notices a painless non-tender 1.5 cm swelling on his left testicle. His general
    practitioner takes some blood tests and identifies a raised serum β-hCG and
    α-fetoprotein.
A Epididymo-orchitis
B Epididymal cyst
C Epididymitis
D Haematoma
E Hydrocoele
F Idiopathic scrotal oedema
G Idiopathic scrotal haemorrhage
H Inguinoscrotal hernia
I Seminoma
J Teratoma
K Testicular torsion
L Torsion of the hydatid of Morgagni
M Varicocele
A
  1. J Teratoma
    Testicular neoplasms commonly present following a minor scrotal injury. The
    patient who self-examines may feel a new lump which he attributes to the injury.
    Commonly, the lesion is pre-existing and was previously too small or asymptomatic
    to be noticed in the absence of regular examination. A raised serum β-human
    chorionic gonadotropin (β-hCG) and α-fetoprotein is diagnostic of a testicular germ
    cell (non-seminomatous) tumour.
    Dunzendorfer U, Jurincic C. Quantification of alpha-fetoprotein and beta-HCG in testis tumor patients.
    Urol Int 1987; 42:248–253.
84
Q
  1. A 27-year-old man has a painless 2 cm swelling in the upper pole of his right
    testicle. It transilluminates and is palpable separate from the testis.
A Epididymo-orchitis
B Epididymal cyst
C Epididymitis
D Haematoma
E Hydrocoele
F Idiopathic scrotal oedema
G Idiopathic scrotal haemorrhage
H Inguinoscrotal hernia
I Seminoma
J Teratoma
K Testicular torsion
L Torsion of the hydatid of Morgagni
M Varicocele
A
  1. B Epididymal cyst
    Epididymal cysts are benign, fluid-filled swellings in the head of the epididymis
    which do not usually require treatment. They are normally distinct from the testicle
    and transilluminate brightly.
    Walsh TJ, Seeger KT, Turek PJ. Spermatoceles in adults: when does size matter? Arch Androl 2007;
    53:345–348.
85
Q
  1. A 4-year-old infant has painless non-tender bilateral scrotal swelling. He has an
    erythematous area extending towards his perineum.
A Epididymo-orchitis
B Epididymal cyst
C Epididymitis
D Haematoma
E Hydrocoele
F Idiopathic scrotal oedema
G Idiopathic scrotal haemorrhage
H Inguinoscrotal hernia
I Seminoma
J Teratoma
K Testicular torsion
L Torsion of the hydatid of Morgagni
M Varicocele
A
  1. F Idiopathic scrotal oedema
    Acute idiopathic scrotal oedema is a relatively rare cause of an acute scrotum. It is characterised by a rapid onset of marked oedema (with or without erythema), without any tenderness. The child is normally apyrexial and all diagnostic tests are
    negative. The aetiology of this condition is unknown although helminth infection
    has been postulated as a cause.

Galejs LE, Kass EJ. Diagnosis and treatment of the acute scrotum. Am Fam Physician 1999;
59:817–824.
Klin B, El-Kholi N, Ashour K, et al. Acute idiopathic scrotal edema in children—revisited. J Pediatr Surg
2002; 37:1200–2

86
Q
  1. A 32-year-old man has a cystic swelling on the upper pole of his right testicle,
    which he finds uncomfortable when cycling to work.
A Analgesia and close observation
B Antibiotics
C Aspiration
D Excision of cyst
E Herniotomy
F Immediate surgical exploration
G Jaboulay procedure
H Liechtenstein repair of inguinal hernia
I Ligation of left testicular vein
J Lord’s procedure
K Testicular vein embolisation
A
  1. D Excision of cyst
    This patient is likely to have an epididymal cyst, although it may represent a
    more sinister lesion. Simple epididymal cysts are normally treated conservatively
    unless they change or the patient becomes symptomatic. In such cases surgery,
    in the form of excision, is recommended. Newer techniques to treat epididymal
    cysts have started to emerge including the use of sclerotherapy under ultrasound
    guidance.
    Pierie S, Agresti P, Morucci M, et al. A therapeutic alternative in the treatment of epididymal cysts:
    percutaneous sclerotherapy. Radiol Med 2003; 105:462–470.
87
Q
  1. A 17-year-old has a 5-hour history of severe pain in his right testicle. It is tender
    and lying high in the scrotum.
A Analgesia and close observation
B Antibiotics
C Aspiration
D Excision of cyst
E Herniotomy
F Immediate surgical exploration
G Jaboulay procedure
H Liechtenstein repair of inguinal hernia
I Ligation of left testicular vein
J Lord’s procedure
K Testicular vein embolisation
A
  1. F Immediate surgical exploration
    This adolescent should be suspected to have a testicular torsion and merits
    immediate surgical exploration. The diagnosis is suspected on history and
    examination findings but should be confirmed by operation. If a viable testicle is
    found it should be untwisted and fixed in that position.
    Classically a testicular torsion presents in infants and adolescents with sudden
    onset severe scrotal pain. The testicle may be tender and lying high in the scrotum.
    The chance of testicular salvage decreases as the duration of torsion increases. A
    torsion less than 6 hours is almost always viable whereas a torsion over 24 hours is
    never viable.
    Williamson RCN Torsion of the testis and allied conditions. Br J Surg 1976; 63:465–476.
    Coggon D, Nelms M. Incidence of testicular abnormalities. Lancet 1984: 747.
    Murphy FL, Fletcher L, Pease P. Early scrotal exploration in all cases is the investigation and intervention of
    choice in the acute paediatric scrotum. Pediatr Surg Int 2006; 22:413–416.
88
Q
  1. A 37-year-old man has an asymptomatic swelling in his left testicle. He describes it
    as ‘a bag of worms’ and on standing there is a noticeable increase in size.
A Analgesia and close observation
B Antibiotics
C Aspiration
D Excision of cyst
E Herniotomy
F Immediate surgical exploration
G Jaboulay procedure
H Liechtenstein repair of inguinal hernia
I Ligation of left testicular vein
J Lord’s procedure
K Testicular vein embolisation
A
  1. K Testicular vein embolisation
    This patient has a varicocoele which is an abnormal dilation of the testicular vein due to
    defective valves, compression or obstruction. Approximately 98% develop on the left
    side because of the 90°angle of insertion of the testicular vein into the left renal vein.
    On the right the drainage is directly into the inferior vena cava. Examination findings are
    likened to a ‘bag of worms’. When patients are symptomatic and desire treatment the
    most common method is by radiological embolisation of the testicular vein.
    Pryor JL, Howards SS. Varicocele. Urol Clin North Am 1987; 14:499–513.
    Kuroiwa T, Hasuo K, Yasumori K, et al. Transcatheter embolization of testicular vein for varicocele testis.
    Acta Radiol 1991; 32:311–314.
89
Q
  1. A 29-year-old woman presents with a 2-day history of right iliac fossa pain. It
    was fairly sudden in onset and radiates through to her back. She has no bowel or
    urinary symptoms and it is 2 weeks since her last period. Ultrasound scan reveals a
    trace of fluid in the pelvis but no other abnormalities.
A Appendicitis
B Appendix mass
C Caecal carcinoma
D Diverticular disease
E Ectopic pregnancy
F Mesenteric adenitis
G Mittelschmerz
H Pelvic inflammatory disease
I Perforated viscus
J Renal colic
K Ruptured ovarian cyst
L Terminal ileal Crohn’s disease
M Urinary tract infection
A
  1. G Mittelschmerz
    Mittelschmerz is mid-cycle lower abdominal or pelvic pain which can occur
    suddenly and may be present for several days. Diagnosis is one of exclusion whereby
    ultrasound scan normally reveals no abnormal findings or a trace of pelvic fluid.
    Krohn PL. Intermenstrual pain (the ‘Mittelschmerz’) and the time of ovulation. BMJ 1949; 7, 1:803–805.
90
Q
  1. An 8-year-old boy complains of a 2-day history of right iliac fossa pain. He is
    apyrexial but is guarding with rebound tenderness. He has had a runny nose for
    the past few days and despite his pain he is hungry and very thirsty. There is a
    strong family history of inflammatory bowel disease.
A Appendicitis
B Appendix mass
C Caecal carcinoma
D Diverticular disease
E Ectopic pregnancy
F Mesenteric adenitis
G Mittelschmerz
H Pelvic inflammatory disease
I Perforated viscus
J Renal colic
K Ruptured ovarian cyst
L Terminal ileal Crohn’s disease
M Urinary tract infection
A
  1. F Mesenteric adenitis
    The family history of inflammatory bowel disease is a distractor here as there are no
    gastrointestinal symptoms. This boy is likely to have mesenteric adenitis secondary
    to his recent upper respiratory viral illness. Mesenteric adenitis commonly mimics
    appendicitis but as this patient is hungry and thirsty, it can be reasonably safe to
    exclude appendicitis as a potential cause.
    Rao PM, Rhea JT, Novelline RA. CT diagnosis of mesenteric adenitis. Radiology 1997; 202:145–149.
91
Q
  1. A 39-year-old man presents with a 4-week history of feeling generally unwell. He
    has low grade pyrexia and is off his food. His clothes feel looser and he suspects
    he has lost one and a half stones in weight. He has not vomited but has been
    diarrhoeal up to 10 times per day. He is tender in the right iliac fossa, where there
    is a palpable mass.
A Appendicitis
B Appendix mass
C Caecal carcinoma
D Diverticular disease
E Ectopic pregnancy
F Mesenteric adenitis
G Mittelschmerz
H Pelvic inflammatory disease
I Perforated viscus
J Renal colic
K Ruptured ovarian cyst
L Terminal ileal Crohn’s disease
M Urinary tract infection
A
  1. L Terminal ileal Crohn’s disease
    Crohn’s disease can affect any part of the gastrointestinal tract from the mouth
    to the anus. The main symptoms of terminal ileal Crohn’s disease are diarrhoea,
    abdominal pain and weight loss. The inflammation can lead to the development of a
    mass and malaise, lethargy, anorexia, nausea and vomiting may also feature.
    Shariff U, Narula H, Speake W, Brown S. Terminal ileal Crohn’s disease: conservative surgeon and aggressive
    physician? Colorectal Dis 2009; 11:522–523.
92
Q
  1. A 62-year-old man is the restrained driver in a head-on collision with another car
    at approximately 50 miles per hour. He complains of right-sided chest and thigh
    pain. He is alert but uncooperative with multiple superficial grazes. His pulse is 140
    beats per minute, blood pressure is 100/74mmHg. His trachea is deviated to the
    left and he has a hyperresonant right hemithorax. His abdomen is soft but there is
    some bruising in the right upper quadrant.
A Chest radiograph
B CT of chest, abdomen and pelvis
C CT of head
D Decompression of tension
pneumothorax
E Diagnostic peritoneal lavage
F Intercostal chest drain
G Intravenous access and fluid
resuscitation
H Laparotomy
I Pericardiocentesis
J Skull radiograph
K Thoracotomy
L Ultrasound of the abdomen
A
  1. D Decompression of tension pneumothorax
    In this patient, the hyper-resonance and tracheal deviation is diagnostic of a tension
    pneumothorax. This occurs when the air leak into the pleural space increases to
    the point, where it pushes the mediastinum (including trachea) to the opposite
    hemithorax. There can be pressure on the cava with a massive decrease in cardiac
    output. The treatment here is emergency needle thoracostomy to decompress the
    thorax prior to insertion of a formal intercostal chest drain.
    Barton ED. Tension pneumothorax. Curr Opin Pulm Med 1999; 5:269.
93
Q
  1. A 27-year-old man is brought to the emergency department resuscitation room
    following a high speed road traffic accident. He is in asystole and a focused
    assessment with sonography for trauma (FAST) scan suggests a pericardial
    effusion and some free fluid in the abdomen. Pericardiocentesis is performed and
    a small volume of blood is aspirated. His clinical condition does not improve. He is
    also noticed to have a tense abdomen.
A Chest radiograph
B CT of chest, abdomen and pelvis
C CT of head
D Decompression of tension
pneumothorax
E Diagnostic peritoneal lavage
F Intercostal chest drain
G Intravenous access and fluid
resuscitation
H Laparotomy
I Pericardiocentesis
J Skull radiograph
K Thoracotomy
L Ultrasound of the abdomen
A
  1. K Thoracotomy
    In a trauma situation a positive pericardiocentesis is an indication for an emergency
    thoracotomy. Although a pericardiocentesis may have some benefit, it is essentially
    a diagnostic and temporizing procedure. The cause of the bleeding in the
    pericardial space needs to be addressed and this requires a thoracotomy. In the
    above patient’s case, the positive focused assessment with sonography for trauma
    (FAST) scan for pericardial blood should have been enough to have mandated a
    thoracotomy, although the pericardial decompression may have been a lifesaving
    measure.
    Cothren CC, Moore E. Emergency department thoracotomy for the critically injured patient: Objectives,
    indications, and outcomes. World Journal of Emergency Surgery 2006, 1:4.
94
Q
  1. A 45-year-old male intravenous drug abuser is stabbed in the epigastric region
    with a breadknife. On arrival to the emergency department, he is hypotensive with
    a thread pulse of 160 beats per minute. His SpO2
    is maintained at 95% on 15L 100%
    O2 and he has two large-bore Venflon catheters inserted with fast resuscitation
    fluid. There is a 3 cm incision midway between his umbilicus and xiphisternum
    with visible omentum. During FAST scanning, he deteriorates and becomes
    haemodynamically unstable.
A Chest radiograph
B CT of chest, abdomen and pelvis
C CT of head
D Decompression of tension
pneumothorax
E Diagnostic peritoneal lavage
F Intercostal chest drain
G Intravenous access and fluid
resuscitation
H Laparotomy
I Pericardiocentesis
J Skull radiograph
K Thoracotomy
L Ultrasound of the abdomen
A
  1. H Laparotomy
    Patients with a penetrating injury who are haemodynamically unstable require
    immediate operation. This includes those patients who are ‘non-responders’ and those who only transiently respond to fluid bolus administration. There should not
    be a delay for unnecessary investigations or interventions.
    In junctional injuries (such as chest/abdomen) a decision must be made on which
    cavity to expose first. If at laparotomy there is no identified source of bleeding a
    thoracotomy should be performed (and vice versa). In the above patient’s case, with
    exposed omentum the peritoneum has clearly been breached and the first course of
    action is to perform a laparotomy.
    Biffl WL. Management guidelines for penetrating abdominal trauma. Current Opin Crit Care 2010;
    16:609–617.
    Henneman PL. Penetrating abdominal trauma. Emerg Med Clin North Am 1989; 7:647–666.
95
Q
  1. A 60-year-old man with liver disease is assessed using the Child–Pugh score. His bilirubin is 44 μmol/L. The International normalised ratio is 1.0 and albumin is 40 g/L. He has no ascites or hepatic encephalopathy.
A 1
B 2
C 3
D 4
E 5
F 6
G 7
H 8
I 9
J 10
K 11
A
95. F 6
The Child–Pugh classification uses five variables (albumin, bilirubin, international normalised ratio, severity of ascites and encephalopathy), and each is scored
between one (most normal) and three (most abnormal). The sum of the five scores generates the Child–Pugh score ranging from 5–15 (Table 1.9). The score corresponds to a Child–Pugh grade of A, B or C which can be used to classify the prognosis for the patient.
The patient in this question will score 1 for all of the above fields except for bilirubin (2). This gives a total score of 6 which is equivalent to Child–Pugh class A (Table 1.10). 

Child CG, Turcotte JG. Surgery and portal hypertension. The liver and portal hypertension. Philadelphia: Saunders. 1964: 50–64.
Pugh RN, Murray-Lyon IM, Dawson JL, et al Transection of the oesophagus for bleeding oesophageal varices.
Br J Surg 1973; 60:646–649.

96
Q
  1. A 49-year-old man with acute pancreatitis is assessed during the first 48 hours in hospital and his Imrie score is calculated. He has normal blood tests and gases apart from a white cell count of 14 × 109/L, blood glucose of 14 mmol/L and urea of 15 mmol/L.
A 1
B 2
C 3
D 4
E 5
F 6
G 7
H 8
I 9
J 10
K 11
A

A 1
This patient has an Imrie score of 1, because of his elevated blood glucose.
The Imrie score is a modification of the Glasgow criteria, used to assess the severity
of acute pancreatitis. It is a composite of eight separate parameters, each scoring 1 if
positive. A total score > 3 in the first 48 hours is predictive of severe pancreatitis and
is associated with a worse prognosis.
By adjusting the titles of the measured parameters a useful acronym can be used,
‘PANCREAS’:
Partial pressure oxygen < 8.0 kPa
Age > 55 years
Neutrophilia: white blood cell count >15 × 109/L
Calcium < 2.0 mmol/L
Renal: urea > 16 mmol/L
Enzymes: lactate dehydrogenase > 600 U/L
Albumin < 32 g/L
Sugar: glucose > 10 mmol/L

97
Q
  1. A 64-year-old man is admitted with melaena and haematemesis. His past medical history includes ischaemic heart disease and type 2 diabetes. At presentation he is clammy with a tachycardia of 120 beats per minute and his blood pressure is 92/58 mmHg. He undergoes urgent endoscopy which identifies a Mallory–Weiss tear which has stopped bleeding. His risk of further bleeding and an adverse outcome is assessed using the Rockall score.
A 1
B 2
C 3
D 4
E 5
F 6
G 7
H 8
I 9
J 10
K 11
A
  1. E 5
    The Rockall scoring system (Table 1.11) uses the above criteria to identify patients at risk of adverse outcome following an upper gastrointestinal bleed. The clinical
    features (age, shock and comorbidity) and endoscopic findings (active bleeding and pathological cause) generates a score between 0 and 11.
98
Q
98. A 58-year-old man with stage 5 chronic kidney disease develops painful purple skin lesions with small areas of necrosis on both shins. All pedal pulses are present and there is no venous incompetence.
A Acanthosis nigricans
B Basal cell carcinoma
C Calciphylaxis
D Dermatomyositis (lung cancer)
E Intradermal naevus
F Junctional naevus
G Lentigo maligna
H Keratoacanthoma
I Nodular melanoma
J Seborrhoeic keratosis
K Squamous cell carcinoma L Strawberry naevus
M Thrombophlebitis migrans
A
  1. C Calciphylaxis
    Patients with end stage renal failure may develop calciphylaxis which is deposits of calcium in the small and medium sized vessels. These can present as painful plaques with surrounding purpura which can progress to ischaemic skin lesions with areas of necrotic skin. Necrotic components are typically dark bluish purple or completely black and leathery.
    Nigwekar SU, Wolf M, Sterns RH, Hix JK. Calciphylaxis from nonuremic causes: a systematic review. Clin J Am Soc Nephrol 2008; 3:1139–43.
    Wilmer WA, Magro CM: Calciphylaxis: emerging concepts in prevention, diagnosis, and treatment. Semin Dial 2002; 15:172–186.
99
Q
  1. A 45-year-old man is diagnosed with stomach cancer and he is also noted to have dark thickened skin in several of his body folds and creases.
A Acanthosis nigricans
B Basal cell carcinoma
C Calciphylaxis
D Dermatomyositis (lung cancer)
E Intradermal naevus
F Junctional naevus
G Lentigo maligna
H Keratoacanthoma
I Nodular melanoma
J Seborrhoeic keratosis
K Squamous cell carcinoma
L Strawberry naevus
M Thrombophlebitis migrans
A
  1. A Acanthosis nigricans
    Acanthosis nigricans is a dermatological condition which presents with
    hyperpigmented, thickened skin, which is most marked over body flexures. It is
    associated with malignancy and in a study of 277 cases (by Rigel et al) 56% had
    gastric carcinomas in 55.5%, 18% had another form of intra-abdominal malignancy
    and 27% had various other malignancies.
    Rigel DS, Jacobs MI. Malignant acanthosis nigricans: a review. J Dermatol Surg Oncol 1980; 6:923–927.
100
Q
  1. A 69-year-old woman describes a discoloured patch on her left cheek, which she
    has not noticed any change in approximately 15 years. It has an irregular border
    and is thickened and pigmented with central nodularity.
A Acanthosis nigricans
B Basal cell carcinoma
C Calciphylaxis
D Dermatomyositis (lung cancer)
E Intradermal naevus
F Junctional naevus
G Lentigo maligna
H Keratoacanthoma
I Nodular melanoma
J Seborrhoeic keratosis
K Squamous cell carcinoma
L Strawberry naevus
M Thrombophlebitis migrans
A
  1. G Lentigo maligna
    Lentigo maligna is a melanoma in situ which is normally found as a slow growing
    pigmented lesion on sun exposed areas. It is considered a precancerous lesion which
    grows slowly over many years. The lifetime risk of malignant transformation ranges
    from 2.2–4.7%.
    Cohen, LM. Lentigo maligna and lentigo maligna melanoma. J Am Acad Dermatol 1996; 33:923–936.
101
Q
  1. A weak diuretic associated with hyperkalaemia.
A Adenosine
B Adrenaline
C Amiloride
D Aspirin
E Atenolol
F Bendroflumethiazide
G Digoxin
H Dobutamine
I Dopamine
J Furosemide
K Metolazone
L Noradrenaline
M Simvastatin
A
  1. C Amiloride
    Amiloride (like spironolactone) is a potassium sparing diuretic which blocks the
    sodium channels in the distal convoluted tubules and collecting ducts, thereby
    inhibiting sodium reabsorption. Salt and water are lost in the urine, whereas
    potassium is retained and this may result in hyperkalaemia.

McNay JL, Oran E. Possible predisposition of diabetic patients to hyperkalemia following administration of
potassium-retaining diuretic, amiloride (MK 870). Metabolism. 1970; 19:58–70.

102
Q
  1. A medication which is absolutely contraindicated in pregnancy.
A Adenosine
B Adrenaline
C Amiloride
D Aspirin
E Atenolol
F Bendroflumethiazide
G Digoxin
H Dobutamine
I Dopamine
J Furosemide
K Metolazone
L Noradrenaline
M Simvastatin
A
  1. M Simvastatin
    Statins are absolutely contraindicated in pregnancy because of the suspected risk
    to the fetus. As statins inhibit cholesterol production (which is required for fetal
    development) there is a risk of teratogenesis and the development of the VACTERL
    syndrome. However, other groups have suggested, there is no evidence that exposure
    to simvastatin increase the rate of congenital abnormalities in pregnant women.
    Edison RJ, Muenke M. Mechanistic and epidemiologic considerations in the evaluation of adverse birth
    outcomes following gestational exposure to statins. Am J Med Genetics 2004; 131:287–298.
    Pollack PS, Shields KE, Burnett DM, et al. Pregnancy outcomes after maternal exposure to simvastatin and
    lovastatin. Birth Defects Res A Clin Mol Teratol 2005; 73:888–896.
103
Q
  1. A sympathomimetic with mainly α-adrenergic activity
A Adenosine
B Adrenaline
C Amiloride
D Aspirin
E Atenolol
F Bendroflumethiazide
G Digoxin
H Dobutamine
I Dopamine
J Furosemide
K Metolazone
L Noradrenaline
M Simvastatin
A
  1. L Noradrenaline
    The main categories of adrenergic receptors relevant to vasopressor activity are the
    α1-, β1-, and β2-adrenergic receptors and the dopamine (δ) receptors. Dopamine has
    varying effects on receptors depending upon the dose used (Table 1.12).
    • α1-adrenergic agonism leads to peripheral arteriolar vasoconstriction.
    • β1-adrenergic agonism leads to increased heart rate and force of contractility.
    • β2-adrenergic agonism leads to bronchial smooth muscle and skeletal muscle
    relaxation.
    • (δ) dopaminergic agonism leads to increased renal blood flow at low doses. At
    higher doses there are also cardiac and vasoconstrictive effects.
    Müllner M, Urbanek B, Havel C, et al. Vasopressors for shock. Cochrane Database Syst Rev 2004; CD003709.
104
Q
  1. A 48-year-old woman has recently undergone treatment for hepatocellular
    carcinoma and a tumour marker is used to assess her response to treatment.
A 5-hydroxyindoleacetic acid
B α-fetoprotein
C Bence-Jones protein
D Cancer antigen (CA) 15-3
E Cancer antigen (CA) 19-9
F Cancer antigen (CA) 27.29
G Cancer antigen (CA) 125
H Calcitonin
I Carcinoembryonic antigen
J Human epidermal growth factor
receptor 2 (HER-2)
K Lactate dehydrogenase
L Prostate specific antigen
M Human chorionic gonadotrophin
N Thyroglobulin
A
  1. B α-fetoprotein
    Serum α-fetoprotein (AFP) can be used to diagnose, and guide the treatment of
    hepatocellular carcinoma (HCC). Normal serum values are under 10 ng/mL, but
    they can be elevated in hepatitis to values as high as 100 ng/mL. In previously
    well individuals a serum AFP > 400 ng/mL is diagnostic of HCC. However, in those with ongoing liver disorders (e.g. hepatitis) a value of > 4000 ng/mL is required to
    diagnose HCC.
    AFP is useful in assessing the response to treatment for HCC. Following complete
    tumour removal, the AFP level should normalise. If it subsequently elevated it
    suggests a tumour recurrence.
    Zhou L, Liu J, Luo F. Serum tumor markers for detection of hepatocellular carcinoma. World J
    Gastroenterol 2006; 12:1175–1181.
105
Q
  1. A 62-year-old woman with obstructive jaundice has an elevated marker for
    pancreatic cancer.
A 5-hydroxyindoleacetic acid
B α-fetoprotein
C Bence-Jones protein
D Cancer antigen (CA) 15-3
E Cancer antigen (CA) 19-9
F Cancer antigen (CA) 27.29
G Cancer antigen (CA) 125
H Calcitonin
I Carcinoembryonic antigen
J Human epidermal growth factor
receptor 2 (HER-2)
K Lactate dehydrogenase
L Prostate specific antigen
M Human chorionic gonadotrophin
N Thyroglobulin
A
  1. E Cancer antigen (CA) 19-9
    This assay was originally developed to diagnose colorectal cancer, but in current
    practice it is more commonly used for patients with pancreatic cancer. The upper
    limit of normal for the CA19-9 assay is 37 U/mL and in the early stages of pancreatic
    cancer the blood levels are often normal, so it is not a useful screening test. The
    overall sensitivity of this assay is ~80% with a specificity of approximately 90%. When
    tumours generate blood levels of CA19-9 > 1000 U/mL it is often an indication that
    the tumour is unfortunately unresectable.
    Steinberg W. The clinical utility of the CA 19-9 tumor-associated antigen. Am J Gastroenterol 1990; 85(4):
    350–355.
106
Q
  1. A 23-year-old man with a painless left testicular swelling has an urgent review
    where tumour markers are taken. One of these is strongly positive and he is
    diagnosed with a pure seminoma.
A 5-hydroxyindoleacetic acid
B α-fetoprotein
C Bence-Jones protein
D Cancer antigen (CA) 15-3
E Cancer antigen (CA) 19-9
F Cancer antigen (CA) 27.29
G Cancer antigen (CA) 125
H Calcitonin
I Carcinoembryonic antigen
J Human epidermal growth factor
receptor 2 (HER-2)
K Lactate dehydrogenase
L Prostate specific antigen
M Human chorionic gonadotrophin
N Thyroglobulin
A
  1. M Human chorionic gonadotrophin
    Three tumor markers are used in men with testicular germ cell tumors; the beta
    subunit of human chorionic gonadotropin (β-hCG), α-fetoprotein (AFP) and lactate
    dehydrogenase (LDH). The most commonly used markers are AFP and β-hCG. AFP
    is never elevated in pure seminomas and β-hCG is only elevated in approximately a
    quarter of cases. AFP and β-hCG are both elevated in about 80% of non-seminomatous
    germ cell tumors (NSGCTs). LDH is not used as a diagnostic test but high levels are
    predictive of decreased survival.
    Laguna MP, Pizzocaro G, Klepp O, et al EAU Working Group on Oncological Urology. EAU guidelines on
    testicular cancer. Eur Urol 2001; 40(2):102–10.
    Weissbach L, Bussar-Maatz R. HCG-positive seminoma. Eur Urol 1993; 23 (Suppl) 2:29–32.
107
Q
  1. A newborn term boy with a prenatal diagnosis of Down’s syndrome has profuse
    bilious vomiting. There is a double bubble gas pattern on plain abdominal X-ray.
A Duodenal atresia
B Duplication cyst
C Exomphalos
D Gastroschisis
E Hirschsprung’s disease
F Intussusception
G Malrotation
H Meckel’s diverticulum
I Meconium ileus
J Necrotising enterocolitis
K Pyloric stenosis
L Tracheo-oesophageal fistula
A
  1. A Duodenal atresia
    Duodenal atresia is a failure of embryological development which results in the
    closure, or absence, of part of the duodenum. It occurs in 1;2500 live births and
    infants normally present with early onset of (commonly bilious) vomiting. The
    diagnosis of duodenal atresia can be confirmed by plain abdominal radiography
    or ultrasound which demonstrates the classical ‘double bubble sign’. There is a
    significant association with Down’s syndrome, as approximately 20–40% of all infants
    with duodenal atresia have Down’s syndrome and approximately 8% all infants with
    Down’s syndrome have duodenal atresia. Correctional surgery is always necessary.
    Nixon HH. Duodenal atresia. Br J Hosp Med 1989; 41:134–140.
    Brimblecombe FS, Moore HD. Duodenal atresia and stenosis in infants. Ann Surg 1970; 172:991–995.
    Joppich I, Kellnar S. Surgery of atresia of the gastrointestinal tract. Chirurg 1996; 67(6):576–83.
108
Q
  1. A newborn girl with suspected cystic fibrosis has abdominal distension and
    bilious vomiting.
A Duodenal atresia
B Duplication cyst
C Exomphalos
D Gastroschisis
E Hirschsprung’s disease
F Intussusception
G Malrotation
H Meckel’s diverticulum
I Meconium ileus
J Necrotising enterocolitis
K Pyloric stenosis
L Tracheo-oesophageal fistula
A
  1. I Meconium ileus
    Cystic fibrosis (CF) is an autosomal recessive disorder which presents with meconium
    ileus in 10–20% of patients. The highly viscous meconium can lead to intestinal
    obstruction which is suggestive of the diagnosis of CF. The majority of infants (90%)
    presenting with meconium ileus are subsequently diagnosed with CF.
    Del Pin CA. Management and survival of meconium ileus. A 30-year review. Ann Surg 1992; 215:179–185.
109
Q
  1. A 2-day-old neonate has been vomiting for approximately 12 hours. She has not
    passed any meconium since delivery. After insertion of a finger into the rectum,
    there is an explosive release of gas and diarrhoea.
A Duodenal atresia
B Duplication cyst
C Exomphalos
D Gastroschisis
E Hirschsprung’s disease
F Intussusception
G Malrotation
H Meckel’s diverticulum
I Meconium ileus
J Necrotising enterocolitis
K Pyloric stenosis
L Tracheo-oesophageal fistula
A
  1. E Hirschsprung’s disease
    Hirschsprung’s disease is congenital aganglionosis of the distal colon as a
    consequence of abnormal faetal development and neural crest migration. The
    myenteric (Auerbach) plexus and the submucosal (Meissner) plexus are both
    affected which leads to decreased peristalsis in the affected segment. The anus
    is always involved and aganglionosis normally extends proximally for a short
    distance.
    Hirschsprung’s disease should be considered in a newborn with delayed passage
    of meconium, or a child with constipation since birth. Other symptoms include
    abdominal distension and vomiting. Most infants pass meconium by 24 hours and if
    there is none by 48 hours, a rectal examination should be performed. There may be
    explosive stools after an examining finger is gently placed in the rectum. Definitive
    diagnosis is normally made by performing a rectal suction biopsy.
    Coran AG, Teitelbaum DH. Recent advances in the management of Hirschsprung’s disease. Am J Surg
    2000; 180:382–387.
    Whitehouse FR, Kernohan JW. The myenteric plexus in congenital megacolon. Arch Int Med 1948; 82:75.
110
Q
  1. A visual method used in meta-analysis to demonstrate the relative strength of
    treatment effects in scientific studies examining the same question.
A Coefficient of variation
B Confidence interval
C Forest plot
D Funnel plot
E Histogram
F Likelihood ratio
G Negative predictive value
H p-value
I Positive predictive value
J Power
K Sensitivity
L Specificity
M Standard deviation
N Standard error
A
  1. C Forest plot
    A Forest plot is a visual method used in meta-analysis to display the relative strength of
    treatment effects in multiple studies examining the same specific question. It normally
    has a left hand column containing the names of the studies (in chronological order)
    and a right hand column which displays the treatment effect as an odds ratio. A square
    with horizontal lines is used to represent the odds ratio and the confidence interval
    (this is normally numerically represented adjacent to the diagram). The size of each
    square represents its weight in the overall analysis (Figure 1.16).
    A vertical line representing no effect (at odds ratio =1) is plotted. A cumulative metaanalysis
    line is also plotted (below as a dotted line). The meta-analysed measure of
    effect is plotted as a diamond where the points represent the confidence intervals.
    Lewis S, Clarke M. Forest plots: trying to see the wood and the trees. BMJ 2001; 322:1479.
    Kirkwood RB, Sterne JC. Essential medical statistics, 2nd edn. Oxford: Wiley-Blackwell, 2003: 376–377.
111
Q
  1. The chance of developing a type I error.
A Coefficient of variation
B Confidence interval
C Forest plot
D Funnel plot
E Histogram
F Likelihood ratio
G Negative predictive value
H p-value
I Positive predictive value
J Power
K Sensitivity
L Specificity
M Standard deviation
N Standard error
A
  1. H p-value
    A type I error is a false positive. In statistical terms this means the test identifies a
    difference or a finding, when none actually exists. The rate of this error is noted as ‘α’,
    and is equivalent to the statistical significance.
    In studies a p-value is quoted as the significance level, or the chance of a type I error.
    In layman’s terms when interpreting a p-value it is best to think of it as the odds that
    the test would have produced the same result through chance alone.
    Grunkemeier GL, Wu YX, Furnary AP. What is the Value of a p Value? Ann Thorac Surg 2009; 87:1337–1343.
    Kirkwood RB, Sterne JC. Essential medical statistics, 2nd edn. Oxford: Wiley-Blackwell, 2003: 376–377.
112
Q
  1. A measure used to compare the ratio of whether a given test result would be
    expected in a patient with the target disorder under investigation, compared to
    the chance the same result would be expected in a patient without the target
    disorder.
A Coefficient of variation
B Confidence interval
C Forest plot
D Funnel plot
E Histogram
F Likelihood ratio
G Negative predictive value
H p-value
I Positive predictive value
J Power
K Sensitivity
L Specificity
M Standard deviation
N Standard error
A
  1. F Likelihood ratio
    A likelihood ratio (LR) is a measure of diagnostic accuracy and normally refers to a
    test or an investigation. When considering a clinical finding, the LR is the probability
    of that finding in patients with disease, divided by the probability of the same
    finding in patients without disease:
    LR =
    Probability of finding in patients with disease
    Probability of finding in patients without disease
    For example, in patients with suspected diverticulitis, if the physical sign of ‘left iliac
    fossa (LIF) tenderness’ is present in 90% of patients with confirmed diverticulitis, and
    30% without diverticulitis (i.e. because of urinary sepsis, pelvic inflammatory disease,
    etc.), then the LR for ‘LIF tenderness’ in detecting diverticulitis is 3.0 (90% divided by 30%).
    A LR > 1 suggests strongly for the diagnosis of interest and the greater the number,
    the more convincingly the finding suggests that disease. LR < 1 argues against the
    diagnosis of interest and the closer to 0, the less likely the disease.
    McGee S. Simplifying likelihood ratios. J Gen Int Med 2002; 17: 647–650.
    Kirkwood RB, Sterne JC. Essential medical statistics, 2nd edn. Oxford: Wiley-Blackwell, 2003: 376–377.
113
Q
  1. A surgeon hypothesises that patients presenting with incisional hernias from
    laparotomy wounds at 1 year postoperative are more likely to have body mass
    index > 25.
A Case–control study
B Case report
C Environmental exposure study
D Feasibility trial of intervention
E Longitudinal cohort series
F Meta-analysis
G Nested case–control trial
H Randomised controlled trial
I Systematic review
A
  1. A Case–control study
    There is a population of patients with an adverse outcome, and knowledge of
    the hypothesised risk factor. Although a longitudinal series would provide firm
    evidence of causal relationship, it would be possible to test his hypothesis by
    case-matching patients with, and without incisional hernias at 1 year postoperative,
    and constructing odds ratios relative to body mass index data in 2 x 2 tables.
    Straus SE, Glasziou P, Richardson WS, Haynes RB. Evidence-based medicine: how to practice and teach
    it, 4th edn. London: Churchill Livingstone, 2010.
114
Q
  1. A research registrar is increasingly convinced that laparoscopic colonic resections
    are yielding improved patient outcomes in terms of length of stay and pain scores.
    He is aware of a number of studies which have reported these outcomes. He would
    like to mathematically model the cumulative outcomes data.
A Case–control study
B Case report
C Environmental exposure study
D Feasibility trial of intervention
E Longitudinal cohort series
F Meta-analysis
G Nested case–control trial
H Randomised controlled trial
I Systematic review
A
  1. F Meta-analysis
    This describes an opportunity to perform a meta-analysis of available accrued trial data,
    with the cumulative numerical output being standard approach for meta-analysis.
    Straus SE, Glasziou P, Richardson WS, Haynes RB. Evidence-based medicine: how to practice and teach it,
    4th edn. London: Churchill Livingstone, 2010.
115
Q
  1. A young consultant is appointed to establish laparoscopic hepatic resection service
    in his hospital. After 3 years, he has meticulous data to demonstrate patient risk
    factors and favourable surgical outcomes. He wishes to write up his work.
A Case–control study
B Case report
C Environmental exposure study
D Feasibility trial of intervention
E Longitudinal cohort series
F Meta-analysis
G Nested case–control trial
H Randomised controlled trial
I Systematic review
A
  1. E Longitudinal cohort series
    This scenario describes an opportunity to demonstrate the value of rigourous,
    longitundinal data in adding to the evidence base. Often, data pertaining to ‘real
    life’ practice, and effectiveness, rather than overall efficacy data generated from
    randomised controlled trials, are highly useful and applicable to daily practice,
    and should not be overlooked in generating levels of evidence in guidelines and
    recommendations (Table 1.13).
116
Q
  1. A surgeon has been gathering data on laparoscopic cholecystectomy (patient
    details and outcomes) for a number of years. He would like to know if men or
    women are more likely to have had readmissions with specific complications
    postoperatively. He has approximately 300 patients on his database.
A χ2 test
B Fisher’s exact test
C Kruskal–Wallis test
D Linear regression
E Mann–Whitney U test
F McNemar test
G Multi-logistic regression
H Student’s t-test
I Wilcoxon sign rank test
A
  1. A χ2 test
    This scenario is going to compare two categorical groups (men, women) by a
    specific categorical outcome (readmission: yes/no). This requires a comparison
    of proportions in a 2 x 2 table. If numbers were very small, the Fisher’s exact test
    should be used instead. Standard statistical programmes usually compute both test
    statistics in the appropriate setting, and the output advises which one should be
    applied based on variance.
117
Q
  1. A colorectal surgeon is interested in technology appraisal and wants to define the
    effectiveness of his own practice, comparing open and laparoscopic approaches
    for malignant disease. He chooses lymph node yield as a comparative outcome
    measure.
A χ2 test
B Fisher’s exact test
C Kruskal–Wallis test
D Linear regression
E Mann–Whitney U test
F McNemar test
G Multi-logistic regression
H Student’s t-test
I Wilcoxon sign rank test
A
  1. E Mann–Whitney U test
    In this scenario two categories of patients, laparoscopic and open surgery, are
    to be compared by a continuous outcome (which is unlikely to have a normal
    or parametric distribution). A non-parametric test comparing location of central
    tendency (median and interquartile range); the Mann-Whitney U test should be
    used. This is the non-parametric equivalent of the t-test. If data had been paired (i.e.
    patients matched by sex, tumour stage, and only varying by operative approach),
    then the Wilcoxon sign rank test would need to be used instead.
118
Q
  1. A surgeon believes that patients’ pain following inguinal hernia repair is
    dependent on the mode of surgery (open under general anaesthetic, open
    under local anaesthetic, transabdominal preperitoneal or total extraperitoneal
    laparoscopic techniques). The other surgeons in his unit agree to share their
    patient data, of which there are good numbers for comparisons. Standard
    postoperative pain scores (0–10) have been collated for every patient prior to
    discharge.
A χ2 test
B Fisher’s exact test
C Kruskal–Wallis test
D Linear regression
E Mann–Whitney U test
F McNemar test
G Multi-logistic regression
H Student’s t-test
I Wilcoxon sign rank test
A
  1. C Kruskal–Wallis test
    This third scenario compares a continuous variable between four categories of patients.
    The continuous variable outcome is likely to exhibit highly skewed distribution. It is
    safest to assume the data is not normally distributed and use non-parametric testing.
    The Kruskal–Wallis test should be used unless the pain data are normally distributed. In
    such an unlikely situation, the ANOVA test should be used. The Kruskal–Wallis test may
    be thought of as the Mann–Whitney U test for more than three groups.